Sunteți pe pagina 1din 85

Autor: Amado Crisógono Castro Chonta 2015-2

CONTENIDO

LAS PROPIEDADES DE LOS GASES SE EXPLICAN MEJOR MEDIANTE LA


TEORÍA CINÉTICO MOLECULAR………………………………………………………………….…..….….3
I) GASES IDEALES………………………………………………………….............5
LEYES DE LOS GASES IDEALES ……………………………………………………………….…………..….6
LEY DE BOYLE………………….…………………………………………………………………………..……..6
LEY DE GAY-LUSSAC Y CHARLES……………………………………………………………………..…….10
PRIMER CASO: LEY DE LAS ISOBÁRICAS…………………………………………..……..….…..10
SEGUNDO CASO: LEY DE LAS ISOMÉTRICAS……………………….…….………………………13
ECUACIÓN DE ESTADO DEL GAS IDEAL………………………………………………………….………..17
MEZCLAS DE GASES IDEALES………………………………………………………………….…………….26
LEY DE LAS PRESIONES PARCIALES DE DALTON…………………………………….………..…26
LEY DE LOS VOLÚMENES PARCIALES DE AMAGAT……………………….…………….……...28
DETERMINACIÓN DE LAS MASAS MOLECULARES DE LAS SUSTANCIAS DE BAJA
TEMPERATURA DE VAPORIZACIÓN…………………………………………………….………….…39
LEY DE GRAHAM DE LA DIFUSIÓN……………………..……………………………….…….………42
II) GASES REALES ………………………………………………….………….…..46
LAS ECUACIONES DE ESTADO............................................................................................... 46
ECUACIONES DE ESTADO DE VDW……….…………………….…………………………………..47
FACTORES DE COMPRESIBILIDAD………………………….……………………………………53
MEZCLAS GASEOSAS DE LOS GASES REALES ………………………..……………..……………………..64
ECUACIÓN DE ESTADO…………………………………………………………………………………….64
CONSTANTE PROMEDIO……………………………………………………………………,,….……….65
FACTOR DE COMPRESIBILIDAD MEDIO…………………………………………………..…….…..65
PROPIEDADES SEUDOCRÍTICAS……………………………………………………………...………..66

BIBLIOGRAFIA ………………………………………………………………………………….……….………..72
TABLA DE FACTORES DE CONVERSIÓN……………..………………………………………………….77
PROBLEMAS ………………………………………………………….…………………………………………….82
ALGUNAS ECUACIONES UTILIZADAS………………………………………..…………………………...71

AMADO DE DIOS (amado_dedios@hotmail.com)[Escriba texto] Paá gina 1


Autor: Amado Crisógono Castro Chonta 2015-2
DIAGRAMAS Y GRÁFICOS:
Página
Diagrama Nº1: Expansión de una masa de gas a temperatura constante…….………..3
Diagrama Nº 2: “Ley de las isotermas del gas ideal de Boyle………………..………..6
Diagrama Nº 3: Representación de las isotermas T1 < T2 < T3 de un Gas Ideal en los
diagramas P vs V y T vs p……………………………………………..……….….….8
Diagrama Nº 3,1: El gráfico de la isoterma en el diagrama P vs. V, para este problema
es: ………………………………………………………………….…………….……..9
Diagrama Nº 4: Ley de las isóbaras de un GI de Jacques Charles….…………….…...10
Diagrama Nº 5: Comportamiento del GI en el gráfico V vs. t…….…………….…….11
Diagrama Nº 6: Las isóbaras de Jacques Charles……………………………….……..12
Diagrama Nº 7: Las Isométricas de Joseph-Louis Gay-Lussac.....................................13
Diagrama Nº 8: Gas Ideal encerrado en un cilindro-émbolo, cuyo
volumen puede variar al mover el émbolo………………….……….17
Diagrama Nº 9: De las presiones parciales de Dalton…………………………….…..26
Diagrama Nº10: El agua se transforma en vapor a 100ºC y 1 atm de presión….……..39
Diagrama Nº11: Ley de efusión de Graham…………………………………………..42
Figura Nº1: Comprobación experimental de la relación Pr/Tr = Cte/Vr para un conjunto
de gases………………………………………………………………………………...54
Figura Nº2: Factor de compresibilidad como función de la temperatura y presión……55
Figura Nº3: Comprensibilidad como una función de la temperatura reducida y presión
reducida………………………………………………………………………………...55
Figura Nº3,6a: Gráfica del factor de compresibilidad generalizado……………………73
Figura Nº3,7: Gráfica del factor de compresibilidad generalizado para presiones
bajas…………………………………………………………………………………….74
Figura Nº3,8: Gráfica del factor de compresibilidad generalizado para presiones término
medio…………….…………………………………….…………………………….…75
Figura Nº3,9: Gráfica del factor de compresibilidad generalizado para presiones
altas……………………………………………………………………………………..75
Figura Nº3,10: Gráfica del factor de compresibilidad generalizado por escalas
especiales…………………………………………………………………………….…76

AMADO DE DIOS (amado_dedios@hotmail.com)[Escriba texto] Paá gina 2


Autor: Amado Crisógono Castro Chonta 2015-2

GASES IDEALES Y GASES REALES

2,0 LAS PROPIEDADES DE LOS GASES SE EXPLICAN BIEN


MEDIANTE LA TEORÍA CINÉTICA MOLECULAR:

Para comprender bien las propiedades de los gases y su comportamiento cuando


cambian las condiciones experimentales; como: ¿Por qué se expande un gas cuando se
calienta a presión constante? ¿O por qué aumentea su presión cuando el gas se
comprime a temperatura constante?; es necesario un modelo que nos ayude a visualizar.
Este modelo se llama Teoría Cinética-molecular TCM), que fue desarrollado a lo largo
de 100 años y que culminó en 1857 cuando Rudolf Clausius (1822-1888) publicó en
forma completa y satisfactoria esta teoría. Diagrama 02: Modelo Cinético de la
materia:

TCM de un GAS
La Teoría Cinética- molecular (que trata de las moléculas en movimiento) se resume en
los enunciados siguientes:
AMADO DE DIOS (amado_dedios@hotmail.com)[Escriba texto] Paá gina 3
Autor: Amado Crisógono Castro Chonta 2015-2
1. Los gases están constituidos generalmente por partículas muy pequeñas llamadas
moléculas o átomos.
2. Las partículas de un gas están en constante movimiento, incesante y caótico
ocupando todo el volumen disponible (V). Estas partículas chocan entre si y con las
paredes del recipiente que los contiene.
3. Generalmente se transfiere energía entre las moléculas durante los choques, pero la
energía cinética media de las moléculas (Ec) no cambian con el tiempo, siempre que
la temperatura del gas permanece constante, es decir los choques son perfectamente
elásticos.
4. La energía cinética media de las moléculas (Ec) es directamente proporcional a la
temperatura absoluta (T); y a una temperatura dada, constante, las moléculas de
todos los gases tienen la misma energía cinética media: (Ec T).
5. La presión de un gas (P) es el resultado de estos choques por unidad área, contra las
paredes del recipiente y tienen el mismo valor sobre toda las paredes.
6. Si se reduce el volumen (V), aumentará el número de choques por unidad de área,
aumentando la presión del gas (P), entonces concluimos que el volumen y la presión

 1
son inversamente proporcionales V  para una misma masa de gas (m) a
 p

temperatura constante (T).


7. Bajo condiciones ordinarias de presión (P) y temperatura (T) el espacio ocupado
propiamente por las moléculas (Vpp) dentro de un volumen gaseoso, es una porción
muy pequeña del volumen total del gas. Por ejemplo una molg de un gas ideal a
condiciones normales de presión y temperatura, CNPT, ocupa un volumen de 22,414
L = 22 414 mL= 22 414 cm 3; y una molg de agua líquida, H 2O(l) , ocupa un volumen
de 18mL=18cm3, que se calcula a partir de la densidad del agua:
1g 1mo lg 1mo lg
m  3
( ) entonces 1molg agua= 18 mL=18cm3
cm 18 g 18cm 3

que al analizar concluimos que el volumen del líquido es despreciable al comparar


con el volumen del gas, y también que el volumen molar y la densidad molar son

1
inversamente proporcionales vm (l )   18cm3 / molg
m

Esto significa que en el gas la distancia de separación entre las moléculas son muy
grandes al comparar con sus diámetros moleculares, entonces las fuerzas de atracción

AMADO DE DIOS (amado_dedios@hotmail.com)[Escriba texto] Paá gina 4


Autor: Amado Crisógono Castro Chonta 2015-2
molecular en el gas se puede considerar despreciable. Esta es una de las propiedades
de los GASES IDEALES.
8. Las moléculas de un gas por estar en constante movimiento, poseen energía cinética,
que depende de la temperatura. Un aumento en la temperatura aumenta la energía
cinética de cada molécula y si se mantiene el volumen constante aumentará la
presión de dicho gas, entonces a mayor temperatura mayor presión para una misma
masa de gas.
9. Las moléculas poseen fuerzas de atracción que trata de mantenerlas juntas y fuerzas
de repulsión que está en función de su energía cinética que trata de dispersarlas.
Estos no tienen el mismo valor para diferentes estados físicos de la materia: en los
sólidos la fuerza de atracción alcanza su máximo valor; en los líquidos las fuerzas de
atracción y repulsión son aproximadamente iguales y en los gases la fuerza de
cohesión alcanza su valor mínimo, por estar las moléculas dotadas del máximo
movimiento (energía cinética).
10. Algunas sustancias que se encuentran como gases 1 atm y 25ºC:
Elementos: H2, N2, O2, O3, F2, Cl2, He, Ne, Ar, Kr, Xe, etc.
Compuestos: HF, HCl, HBr, HI, CO, CO2, NH3, NO, NO2, N2O, SO2, H2S, CH4,
HCN (punto de ebullición es 26ºC), etc.
11. Los gases se comportan de dos maneras, como: A) Gases Ideales y
B) Gases Reales

I) GASES IDEALES
El objetivo es estudiar el comportamiento de los gases como gases ideales: leyes,
postulados y propiedades, y estas se expresan por la ecuación de estado de los gases
ideales.
Los Gases Ideales presentan tres propiedades importantes: primero el volumen propio
de sus moléculas son despreciables con respecto al gran volumen del recipiente que la
ocupan (Vpp=0), y segundo la fuerza de atracción de sus moléculas son despreciables
por que la distancia de separación entre sus moléculas son muy grandes (Fa=0), y
tercero sus moléculas presentan choques elásticos entre sí y con las paredes del
recipiente. Una colisión elástica es aquella en la cual no hay cambio en la energía
cinética total; en consecuencia, las moléculas no absorben energía en los movimientos
internos y las paredes tampoco absorben energía del gas. Las moléculas de los gases

AMADO DE DIOS (amado_dedios@hotmail.com)[Escriba texto] Paá gina 5


Autor: Amado Crisógono Castro Chonta 2015-2
reales se atraen y se repelen entre sí en una pared que se describirán más adelante, y las
reacciones químicas son ejemplos de colisiones inelásticas.
Diagrama 03: Propiedades de los Gases Ideales

Una ecuación de estado relaciona la cantidad molar (n) y el volumen de un gas (V) con
la temperatura (T) y presión (P). La relación más simple y usada de éstas es la ecuación
de estado de los gases ideales (PV=nRT), la cual, aunque es aproximada, resulta
adecuada para muchos cálculos de ingeniería donde participa gases a baja presión y
temperatura moderadamente alta.

2,1 LEYES DE LOS GASES IDEALES:


2, 1, 1 LEY DE BOYLE: Diagrama 04: “Ley de las isotermas”

Llamada también “Ley de las Isotermas” por ser un proceso a TEMPERATURA


CONSTANTE. El sistema termodinámico es el gas ideal que está encerrado dentro
del cilindro-émbolo. En el gráfico observamos si a temperatura constante (T) se

AMADO DE DIOS (amado_dedios@hotmail.com)[Escriba texto] Paá gina 6


Autor: Amado Crisógono Castro Chonta 2015-2
duplica el volumen (V) de una misma masa de gas ideal (m), entonces la presión del
gas (P) disminuye a la mitad y viceversa. Este comportamiento del gas ideal expresa
que el volumen y la presión son inversamente proporcionales para una misma masa

1
de gas a temperatura constante: ( V )T,m y su ecuación termodinámica es:
P
1
V  K B   entonces (V.P)T,m = KB = constante de Boyle…… (1)
P
vemos que el producto de la presión del gas por su volumen para una misma masa
de gas ideal a temperatura constante da como resultado un valor constante, llamada
Constante de Boyle (KB) en honor a su descubridor Robert Boyle. Para cambios
finitos de estado termodinámico de una misma masa de gas ideal a temperatura
constante, si un gas pasa de un estado (1) a un estado (2) tenemos:
Estado 1: (V1P1)T,m = KB y Estado2 : (V2P2)T,m = KB
Para un mismo proceso termodinámico el valor de KB es constante, entonces:
Igualando obtenemos (KB = V1P1 = V2P 2 ) T,m ……………..……. (2)
Para el estado n se tiene (VnPn = KB)T,m …………………...………..… (3)
Igualando las ecuaciones (2) y (3) tenemos:
(V1P1 = V2P2 =… =VnPn= KB)T,m…..............................(4)
Manteniendo la temperatura constante para una misma masa de un gas ideal,
siempre el producto de la presión por su volumen respectivo es un valor constante,
igual a la Constante de Boyle (KB). La ecuación (4) también se puede expresar
como:
V1 P V2 Pn V1 P
(  2 )T,m y (  ) y (  n )T,m ………(5)
V2 P1 Vn P2 T,m Vn P1

También la ecuación de Boyle se puede expresar en función de sus densidades ()


Sea 1 para el estado 1 y 2 para el estado 2 de una misma masa de gas ideal (m) a
temperatura (T) constante:
m  masa
 entonces (m = 1 V1)T es el estado 1
V  volumen

(m = 2 V2)T es el estado 2
Como la masa del gas es la misma y la temperatura es constante, podemos igualar
1 V2
(m = 1V1   2V2 )T  (  ) ……………. (6)
 2 V1 T,m

V2 P  P
 1  1  1
V1 P2 2 P2
AMADO DE DIOS (amado_dedios@hotmail.com)[Escriba texto] Paá gina 7
Autor: Amado Crisógono Castro Chonta 2015-2
Pero ( )T,m…………….(7)

En conclusión para una misma masa de gas a temperatura constante la densidad de


un gas ideal es directamente proporcional a su presión, ecuación (7), e inversamente
proporcional a su volumen, ecuación (6)
Un gas es ideal cuando las fuerzas de cohesión son cero y cuando el volumen propio
de las moléculas es despreciable, es decir cero, frente al volumen total del recipiente
ocupado por el gas. Esta ley no se cumple para los gases reales, salvo a presiones
bajas y temperaturas moderadamente altas.

Diagrama Nº 5: Representación de las isotermas T1 < T2 < T3 de un Gas


Ideal en los diagramas P vs V y T vs P.

P T
_________________P=cte. T3

T3

T2 KB3
T2
T1 KB2
T1

KB1
V
P
(PV)T,m = KB Isotermas de un gas ideal

Las isotermas de un gas ideal en un diagrama P vs V, son asíntotas a los eje de la


presión (P) y del volumen (V). Y son rectas paralelas al eje de la presión en el
diagrama T vs P. Para cada isoterma el valor de la constante de Boyle son diferentes.
PROBLEMA Nº 1: Una masa de gas ideal tiene un volumen de 20 L y una presión
de 750 mmHg. Si la temperatura permanece constante cuando su presión varía
sucesivamente a 600, 300, 200 y 100 mmHg aplicando la ecuación de estado de la
Ley de Boyle trazar su ISOTERMA.
Solución del problema :
1. Calcule primero la constante de Boyle (KB):
(PV = KB)T,m V1 = 20 L P1 = 750mmHg m = constante
T1 = T2 = T3 = T4 = T5 = T = constante
AMADO DE DIOS (amado_dedios@hotmail.com)[Escriba texto] Paá gina 8
Autor: Amado Crisógono Castro Chonta 2015-2
(20L) (750 mmHg) = KB entonces 15 000 L.mmHg = KB
2. Calcule los volúmenes: V2, V3, V4, V5 para cada variación de Presión :
KB 15 000 LmmHg
( V2  )m,T V2 
600mmHg
 25 L
P2
15 000 LmmHg
V3   50 L
300mmHg
15 000 LmmHg
V4   75 L
200mmHg

15 000 LmmHg
V5   150 L
100mmHg

Diagrama Nº 5,1: El gráfico de la isoterma en el diagrama P vs. V, para este


problema es:
P (mmHg)
P(mmHg) V(L)
800 750 20

750 600 25
300 50
600 200 75

500
100 150

400
RESPUESTA:
300
La curva es la isoterma de este problema.

200

100

20 V(L)
25 50 75 100 125 150

PROBLEMA Nº 2: Se desea averiguar la presión adicional en otro que se debe proveer


al gas a la presión de una atmósfera contenida en un globo, para obligarle a llenar otro
globo cuyo volumen es ½ del volumen del primero, con el objeto que al final quede
todo el sistema a la presión de una atmósfera.

Robert Boyle (Lismore, 25 de enero de 1627 - Londres, 30 de diciembre de 1691)


físico y químico irlandes, principalmente conocido por su formulación de la Ley de
Boyle-Mariotte. Es considerado uno de los padres de la química moderna.

AMADO DE DIOS (amado_dedios@hotmail.com)[Escriba texto] Paá gina 9


Autor: Amado Crisógono Castro Chonta 2015-2

http://www.juntadeandalucia.es/averroes/recursos_informaticos/andared02/leyes_g
ases/ley_boyle.html.
http://platea.pntic.mec.es/~cpalacio/boyle2.htm.
http://www.youtube.com/watch?v=GLlivXIIYiY

2, 1, 2 LEYES DE GAY – LUSSAC Y CHARLES


Primer caso: “LEY DE LAS ISÓBARAS” variación del volumen (V) de una misma
masa (m) de gas ideal con la temperatura (T) a presión (P) constante.
Segundo caso: “LEY DE LAS ISÓCORAS O ISOMÉTRICAS” variación de la presión
(P) de una misma masa (m) de gas ideal con la temperatura (T) a
volumen (V) constante.
PRIMER CASO: “LEY DE LAS ISÓBARAS”

Diagrama Nº 6: Ley de las isóbaras de un GI de Jacques Charles.

A presión constante el volumen de una misma masa de cualquier gas ideal aumenta en
1/273,15 = 0,003 66 =  de su volumen a 0ºC, por cada grado centígrado de elevación
en la temperatura.
Sea (V0) el volumen de una determinada masa de gas a 0ºC, y (Vt) su volumen a la
temperatura (t) expresada en grado Celsius, estará dado por la ecuación:
Vt = Vo + V pero V = Vo t
Combinando las dos ecuaciones tenemos: Vt = Vo + Vo t ...................(8)
1
donde   273,15

AMADO DE DIOS (amado_dedios@hotmail.com)[Escriba texto] Paá gina 10


Autor: Amado Crisógono Castro Chonta 2015-2
 t 
Factorizando es Vt = Vo ( 1 + t) y reemplazando tenemos: Vt  Vo 1  273,15 
 

 273,15  t 
Vt  Vo   …………………………….(8a)
 273,15 

Pero 273,15 + t (ºC) =T (K) es temperatura absoluta.

Entonces: Para un estado (1) Para un estado (2)


(273,15  t1 ) ( 273,15  t 2 )
V1  Vo V2  Vo
273,15 273,15
273,15 + t1 = T1 273,15 + t2 = T2
VoT1 VoT 2
V1  V2 
273,15 273,15
dividiendo las ecuaciones de los dos estados termodinámicos y factorizando tenemos:
V1 VoT1 / 273 V T 
   1  1  …………………………..(9)
V2 VoT 2 / 273  V2 T2  P1m

Para una misma masa de un gas ideal en un proceso termodinámico a presión constante,
a mayor temperatura mayor volumen, entonces el volumen (V) y la temperatura (T) son
directamente proporcionales: (V  T)P,m
Su ecuación de estado termodinámico es:

V �

(V = KCh T)P,m ó � �  K Ch ..............(10)
�T �
P ,m

Donde KCh es la constante de Charles en merito a estas leyes descubiertos y estudiados


por el científico Jacques Charles. Ecuación termodinámico para cambios de estado
termodinámico:
V1 V2 Vn
(   ..............   KCh )P,m. ……...(11)
T1 T2 Tn
Representación Gráfica : La ecuación (8) es la ecuación de una recta que corta el eje de
volumen: Vt = Vo + Vo t ...............................................(8)
Si t1= 0  Vt1 = Vo
y Si t2 = -273,15ºC  Vt2 = 0
Donde Vo es una constante que es igual a 22,414 L por cada molg de Gas Ideal (GI) a
la temperatura de 0ºC y presión una atmosfera.
V

AMADO DE DIOS (amado_dedios@hotmail.com)[Escriba texto] Paá gina 11


Autor: Amado Crisógono Castro Chonta 2015-2

(Vo) = Tg Pendiente de la


recta
Vo = 22,414L

0 t ºC
-273,15ºC =0
Diagrama 0ºK
Nº 7: Comportamiento del GI en el gráfico V vs. t.
De la misma manera la representación gráfica de la ecuación de estado de la Ley de
Jacques Charles es función de la temperatura absoluta, y es una línea recta
( V = KChT)m,P…………………………(10)
Vemos que la ecuación (10) no tiene intercepto, y parte del origen de coordenadas en el
diagrama volumen vs temperatura (V vs T).
En el diagrama se expresa rectas isóbaras que nos indica valores de presión constante de
mayor a menor (Pn>P3>P2>P1) y cada recta es un valor de la constante de Jacques
Charles donde KGL1>K GL2>K GL3>K GLn
Diagrama Nº 8: Las isóbaras de Jacques Charles
V(L)
Pn > P3 > P2 > P1

P1
P2
CADA RECTA Pi ES UNA
P3
ISÓBARA
Pn

_____________________V= cte.

T(K)

También se expresa la ecuación de Jacques Charles en función de sus densidades ()


 1 V2 V2 T2  2 T1
(  )m,P y (  )  (  )m,P …... (11)
 2 V1 V1 T1 m,P 1 T2

La densidad de una misma masa de gas a presión constante es inversamente


proporcional a su temperatura absoluta.

Algunos experimentos que pueden demostrar esta ley son los siguientes: En un tubo de
ensayo se deposita un poco de agua y se tapa el tubo con un corcho, luego se empieza a
calentar el tubo con un mechero, el gas que había dentro del tubo (el vapor generado por
el agua y el aire) empezara a expandirse, tanto que necesita una vía de escape, así que
¡pum! el corcho saldrá volando y el gas ya podrá salir tranquilamente.

Para este experimento se necesitara una botella de vidrio, un mechero y un globo de


caucho. En la punta de la botella pondremos la boca del globo y luego calentaremos la
AMADO DE DIOS (amado_dedios@hotmail.com)[Escriba texto] Paá gina 12
Autor: Amado Crisógono Castro Chonta 2015-2
botella. Después de un buen rato el gas se expenderá hasta inflar el globo de caucho. Un
experimento muy contrario a este seria meter un globo inflado totalmente a una nevera,
si con el calor los gases se expanden, con el frío pasa todo lo contrario, así que después
de esperar unas 3 horas veremos el globo un poco desinflado.

Jacques Alexandre César Charles (Beaugency-sur-Loire, 12 de noviembre de 1746 - 7


de abril de 1823) inventor, científico y matemático francés. Fue el primero en realizar
un viaje en globo aerostático, el 27 de agosto de 1783. El 1 de diciembre de ese año,
junto con Ainé Roberts, logró elevarse hasta una altura de 550 metros. Inventó varios
dispositivos, entre ellos un densímetro (también llamado hidrómetro), aparato que mide
la gravedad específica de los líquidos. Cerca del 1787 descubrió la ley de Charles. Su
descubrimiento fue previo al de Louis Joseph Gay-Lussac, que publicó en 1802 la ley
de expansión de los gases. Charles fue electo en 1793 como miembro de la Académie
des Sciences, instituto real de Francia. Fue profesor de física hasta su muerte el 7 de
abril de 1823.

SEGUNDO CASO: “LEY DE LAS ISÓCORAS”

Ley de las ISÓCORAS O ISOMÉTRICAS, llamada también Ley de Gay Lussac,


expresa que a volumen constante la presión de una determinada masa de un gas ideal

AMADO DE DIOS (amado_dedios@hotmail.com)[Escriba texto] Paá gina 13


Autor: Amado Crisógono Castro Chonta 2015-2
aumenta en 1/273,15 = 0,003 66 = , de su presión por cada grado (ºC) de elevación en
su temperatura análogo al 1er. Caso
Po: presión del gas a 0ºC
Pt: presión del gas a una temperatura t (ºC)
Pt = Po + Po t factorizando Pt = Po (1 + t)
 P1 T1   273  t 
   porque Pt  Po 
 P2 T2 V ,m  273 

�P �
� �  KGL (P = KGL T )V,m ………(12)
�T �
V1m

Donde KGL es la constante de Gay-Lussac en merito a estas leyes descubiertos y


estudiados por el científico Joseph-Louis Gay-Lussac
Para una misma masa de gas ideal, a volumen constante, la presión del gas es
directamente proporcional a su temperatura absoluta.
Representación gráfica: En esta ley la presión es función de su temperatura absoluta,
que es una línea recta. Al analizar Vn > V3 > V2 > V1

Diagrama Nº 9: Las Isométricas de Joseph-Louis Gay-Lussac


P (atm)
V1
V2 Observamos que Vn>V3>V2>V1

V3
CADA RECTA Vi ES UNA

Vn
ISÓMÉTRICA
____________________________P=cte

T(K)
Louis Joseph o Joseph-Louis Gay-Lussac (Saint-Léonard-de-Noblat, Francia, 6 de
diciembre de 1778 - París, Francia, 9 de mayo de 1850) fue un químico y físico francés.
Es conocido en la actualidad por su contribución a las leyes de los gases. En 1802, Gay-
Lussac fue el primero en formular la ley según la cual un gas se expande
proporcionalmente a su temperatura (absoluta) si se mantiene constante la presión. Esta
ley es conocida en la actualidad como Ley de Charles.

AMADO DE DIOS (amado_dedios@hotmail.com)[Escriba texto] Paá gina 14


Autor: Amado Crisógono Castro Chonta 2015-2
Los gases reales no siguen con exactitud las leyes de Gay – Lussac, Charles y de Boyle.
Las variaciones son muchos menores cuando el gas real está a alta temperatura y baja
presión. Las desviaciones varían de un gas a otro.

PROBLEMA Nº 3: El aire contenido en una llanta de automóvil presenta una presión


manométrica de 25 lbf/pulg2 medida a 0ºC. ¿Calcular su presión manométrica a 27ºC?
1º Calcular presión y temperatura absoluta en el estado 1:
P = Pman + Po P  presión absoluta
1 atm = 14,7 lbf/pulg2 Pman  presión manométrica
P = (25 + 14,7) lbf/pulg2 Po  presión atmosférica
P1 = 39,7 lbf/pulg2 absoluta T1 = 0ºC + 273,15 => T1 = 273,15
2º Calcular la temperatura y presión en el estado 2:
T2 = 27ºC + 273,15 = 300,15K P2 =?

P1 P2 T 
Ecuación:   P2  P1  2  ; reemplazando valores:
T 1 T2  T1 
lb  300,15 K 
P2  39,7   tenemos que la P2 = 43,6 lb/pulg2 abs
pu lg 2  273,15 K 

3º Calcular la presión manométrica del aire a 27ºC, estado 2:


Pman2 = P2 – Po entonces Pman2 = (43,6 – 14,7) lb/pulg2
Respuesta: La presión manométrica a 27ºC es Pman2 = 28,9 lb/pulg2

PROBLEMA 4: Si se conserva constante el volumen de un gas calcular la temperatura


en grados Celsius cuando se triplica la presión, si la temperatura inicial medida es 50ºC.
1º Calcular la temperatura absoluta en el estado 1: T1 = (50+273,15)K= 323,15 K
2º Calcular la presión en el estado 2: P2 = 3P1
3P
3º Calcular la temperatura en el estado 2: T2  323,15 K ( ) entonces
P
T2 = 969,45 K y en grados Celsius es t(ºC) = 969,45 – 273,15 ;
luego tenemos: t2 = 696,3ºC.
Respuesta: La temperatura final es 696,3ºC

PROBLEMA 5: Un frasco abierto contiene aire a 27ºC y 1atm, si se calienta hasta 87ºC
a la presión constante atmosférica. determinar la fracción del aire que es expulsado.
1º Calcular la temperatura absoluta en el estado inicial (T1) y su volumen V1:

AMADO DE DIOS (amado_dedios@hotmail.com)[Escriba texto] Paá gina 15


Autor: Amado Crisógono Castro Chonta 2015-2
1º K
. T1 = 27ºC x + 273,15K; entonces la temperatura T1 = 300,15K
1º C
P1 = 1 atm
V1 = 10 L valor asumido = V
2º Calcular la temperatura absoluta en el estado final (T2) y su volumen final (V2):
T2 = (87 + 273,15)K = 360,15K Calculando el V2
360,15º K
P2 = P1=1 atm V2 = l0 L 300,15º K  12 L

V2 =? Calculando la fracción de volumen expulsado :


(12  10) L 1

10 L 5
1
Respuesta: La fracción de aire expulsado es 5
V

PROBLEMA 6: Un frasco abierto contiene aire a 20ºC y 1atm; calcular la temperatura


a la que debe calentarse el frasco para que expulse 1/5 del aire que contiene, si se
mantiene la misma presión.
1º Calcular las propiedades termodinámicas del estado 1:
1K
T1 = 20ºC  273,15 K  293,15º K
1º C
P1 = 1 atm y V1 = V0
2º Calcular las propiedades termodinámicas del estado 2:
1 
T2 =? V2   Vo  Vo 
5 
6
P2 = 1 atm V2 = Vo Ecuación: T2 = T1 (V2/V1)
5
6
Vo
T2 = 293,15 K 5
Vo
T2 = 351,78 K – 273,15 K RESPUESTA: El frasco debe calentarse a la temperatura
 T2 = 78,63ºC de 78,6ºC y 1 atm de presión
Problema 7: Una masa de gas a presión constante ocupa un volumen de 20L a una
temperatura de 30ºC. Aplicando la Ley de Gay Lussac calcular lo siguiente: a) el
volumen que el gas ocuparía si la temperatura se elevara a 150ºC; b) la temperatura en
grados Celsius si el volumen cambia hasta medir 5L.
Problema 8: Una masa constante de gas a temperatura constante tiene una presión
manométrica de 1atm en un lugar donde la presión atmosférica es 700 mmHg y ocupa

AMADO DE DIOS (amado_dedios@hotmail.com)[Escriba texto] Paá gina 16


Autor: Amado Crisógono Castro Chonta 2015-2
un volumen de 50L. Aplicando la ley de Boyle calcular lo siguiente: a) el volumen que
el gas ocuparía si la presión absoluta aumentara en 1000 mmHg; b) la presión del gas si
el volumen disminuye hasta medir 10L.
&&&&&&&&&&&&&&&&&&&&&&&&&&&&&&&&&&&&&&&&&&&&&
“Yo te ha amado desde toda la eternidad” Jeremías 31,3.
“Antes de ser formado en el vientre de tu madre, Yo te conocía, y antes que nacieses, Yo te
escogí” Jeremías 1,5
“Te he llamado por tu nombre y tú me perteneces” Isaías 43,1-2
“Tengo tu nombre grabado en la palma de mis manos y no puedo olvidarme de ti” Isaías
49,15-16.
“Todo lo mío es tuyo y todo lo tuyo es mío” San Juan 17,10
“Hasta los pelos de tu cabeza los tengo contados” San Mateo 10,30
“Hijo mío, escucha mis palabras para ser sabio” Proverbios 23,19
“Confía en Mí y no te apoyes en tu propia inteligencia” Prov. 3,5
&&&&&&&&&&&&&&&&&&&&&&&&&&&&&&&&&&
2, 1,3 ECUACIÓN DE ESTADO DEL GAS IDEAL

Diagrama Nº 10: Gas Ideal encerrado en un cilindro y su volumen puede disminuir al


poner una pesa sobre el émbolo.

Para una masa constante la ecuación de estado de un gas ideal será: V = f (P, T)…(13)
Un estado termodinámico se fija mediante los parámetros P, V, T
Estado 1 Estado 2
P1, V1, T1 P2, V2, T2

ISOTERMA ESTADO P = cte


ISÓBARA
T = cte INTERME
Vx, T1, P2

AMADO DE DIOS (amado_dedios@hotmail.com)[Escriba texto] Paá gina 17


Autor: Amado Crisógono Castro Chonta 2015-2
1º Relacionando el estado 1 con el estado intermedio (T1 = cte)
Por la ley de Boyle :
 P1 
Vx, P2 = P1V1  Vx = V1   ………………(13,1)
 P2 
2º Relacionando el estado intermedio con el estado 2 donde la P = cte  por la ley de

 T1 

 Vx  V2  

V V
Charles: (1° caso) x  2  T2  ………… (13,2)
T T
1 2

3º Igualando la ecuación 13,1 con la ecuación 13,2


V1 P1 V T V1 P1 V P
 2 1 ordenando  2 2 = cte. ……………….(14)
P2 T2 T1 T2

La ecuación (14) es la 1ª expresión termodinámica de la ecuación de estado del gas


ideal. En general:
 PV 
   constante = n.R……………………. (15)
 T m

Se ha determinado experimentalmente que el valor de la constante de la ecuación (15)


es igual al producto del número de moles (n) por la constante universal de los gases
ideales (R).
Diagrama 10: Ley de Avogadro

Pero por la Ley de Avogadro: cualquier gas ideal que contenga el mismo número de
moles (n) a las mismas condiciones de P y T ocupan el mismo volumen (V), es decir

volúmenes iguales contienen el mismo número de moles: ( V = KAn)P,T……..(16)


1 mol- moléculas= 1 molg = 6,023 x 1023 moléculas
1 mol-átomo = 1 molg = 6,023 x 1023 átomos

AMADO DE DIOS (amado_dedios@hotmail.com)[Escriba texto] Paá gina 18


Autor: Amado Crisógono Castro Chonta 2015-2
Si el volumen total (V) del gas ideal de la ecuación (15) se cambia por su volumen
molar (Vm) esta se transforma en la ecuación (17) donde la nueva constante es igual a la
constante universal de los gases ideales (R).
 PVm 
  = constante= R  Constante universal de los gases ideales………(17)
 T m

Vm  volumen molar (es decir el volumen de una molg de sustancia)


V
Vm   V  nVm ……. (18) y (Vm)C.N = 22,414 L/mol ………….(18,1)
n

Para “n” moles la ecuación (17) se transforma en la ecuación (15):


P ( nVm )
 nR donde n  número de moles
T
PV
T
 nR ó PV = nRT…………………..(19)
La ecuación (19) es la 2da expresión termodinámica de la ecuación de estado de un gas
ideal, que relaciona n, P, V, T para un único estado termodinámico:
m  masa
Conocemos que el número de moles es: n …….(20)
M  masamolar

Reemplazando la ecuación (20) en la ecuación (19) obtenemos:


m
PV  RT …………..(21)
M
mRT
Para calcular la masa molar (M) de un gas ideal: M  ………(21,1)
PV
MPV
Para calcular la masa (m) de un gas ideal: m …………………(21,2)
RT
Poniendo la ecuación (21) en función de su densidad:
m m  masa
PM    RT …..(21,3) donde la densidad es  …….(22)
V  V  volumen

Reemplazando la ecuación (22) e la ecuación (21,3) obtenemos:


PM = RT despejando obtenemos la ecuación (23) para calcular la densidad de un gas

PM
ideal:  ………………….. (23)
RT
PVm MPV PV
Para los gases ideales las relaciones 1 ó 1 ó 1 ….(24)
RT mRT nRT
de estos parámetros según la ecuación (24) es 1 y para los gases reales es distinta de la
unidad.

AMADO DE DIOS (amado_dedios@hotmail.com)[Escriba texto] Paá gina 19


Autor: Amado Crisógono Castro Chonta 2015-2
PVm
 Z  donde Z se llama coeficiente de comprensibilidad…..(25)
RT
A condiciones normales de presión y temperatura (0ºC y 1 atm) se ha encontrado que
una unidad molg de un gas ideal ocupa un volumen de 22,414 L = 22 414mL.
2, 1, 3a: UNIDADES Y DIMENSIONES DE LA CONSTANTE UNIVERSAL DE
LOS GASES IDEALES (R)

PV
1º Hallando las dimensiones de R ...ecuación (19): las dimensiones de la presión
nT
F
es P   F / L2 y del volumen es V <=> L 3 y del número de moles es nmolg y
A
de la temperatura es T grado, y reemplazando todo en la ecuación (19) obtenemos:

( FL2 )( L3 ) ( FL )
R   …..…..(26), entonces la constante (R) universal
mo lg .grado mo lg .grado

de los gases ideales expresa energía del gas ideal por cada unidad de molg y por cada
grado de temperatura
ENERGIA
R  ……………….(27)
mo lg .grado

2º Determinando las unidades de la constante universal de los gases ideales (R):


a) Expresando R en atm. L A CNPT: P = 1 atm.
(1atm)(22,414 moLlg )
R T = 0ºC + 273 = 273,15 K
grado
Las unidades atm.L son unidades V = 22,414 L
de energía n = 1 molg

(1atm)(22,414 L) atm.L
R � R  0,08206 ………(28)
(1mo lg)(273,15K ) mo lg.K

b) Expresado R en ergios Tomando como base CNPT :


(1,0133x106 dina
cm 2
)(22414cm3 ) dina
R P  1 atm = 1,0133 x 106
(1mo lg)(273,15K ) cm 2

103 cm 3
La unidad ergio es unidad de V  22,414L x
1L

AMADO DE DIOS (amado_dedios@hotmail.com)[Escriba texto] Paá gina 20


Autor: Amado Crisógono Castro Chonta 2015-2
energía. V = 22 414cm3
n = 1 molg
Entonces el valor de (R) es: R = 8,314 x 107 erg/molg.K …………(29)
T = ºC + 273,15 = 273,15 K
c) Expresando R en el SI Tomando como base CNPT
1 julio = 107erg

erg  1 julio 
R  8,314 x10 7  7  entonces R = 8,314 julio /molg.K ….(30)
mo lg .K  10 erg 
d) Expresando R en calorías : tomando como base CNPT
Conocemos que 1 cal = 4,184 julio entonces
julio 1cal cal cal
R  8,314 x factorizando R  1,987  1,99 …
mo lg .K 4,184 julio mo lg .K mo lg .K

(31)
Para muchos cálculos de ingeniería se utiliza el valor de R= 2,00cal/molg.K....(31,1)

PROBLEMA 9: Cierta masa de gas ocupa 200 litros a 95ºC y 782 mmHg ¿Cuál será
el volumen ocupado por dicha masa de gas a 65ºC y 815mmHg?
Solución del problema: 1º Las propiedades de P,V,T en los estados 1 y 2 son:
Estado 1 Estado 2
P1 = 782 mmHg P2 = 815mmHg
T1 = (95 + 273,15)K T2 = (65 + 273,15)K
T1 = 368,15K = 338,15K
V1 = 200L V2 = ¿?
2º Utilizando la ecuación (14) para calcular el volumen en el estado 2:
P1V1 PV  T  PV
 2 2 ….(14) despejando V2   2  1 1 reemplazando valores
T1 T2  P2  T1
tenemos:
338,15 K 782mmHgx200 L
V2  x entonces el volumen 2 es: V2 = 176,3 L
815mmHg 368,15 K

Respuesta: el volumen final ocupado por el gas es 176,3L

PROBLEMA 10: Dentro de un cilindro-pistón está encerrado 1,00 m3 de aire a la


presión de 5 atm y a 20C. ¿calcular:

AMADO DE DIOS (amado_dedios@hotmail.com)[Escriba texto] Paá gina 21


Autor: Amado Crisógono Castro Chonta 2015-2
a) La presión del aire interior, si manteniendo constante la posición del pistón,
se calienta el aire hasta 120ºC? y
b) La presión del aire interior, si luego de haber calentado el aire hasta 120ºC,
se le deja expandir isotérmicamente hasta ocupar un volumen de 5,00m3?
Respuesta a) 6,70 atm b) 1,34atm

PROBLEMA 11: Una masa gaseosa pasa de 20ºC a 150ºC, ocupando a esta segunda
temperatura y a la presión de 600 mmHg un volumen doble del primitivo. ¿Calcular la
presión inicial?
Solución del problema: 1º Las propiedades de P,V,T en los estados inicial (1) y final
(2) son:
Estado inicial (1) Estado final (2)
T1 = (20 + 273,15)K T2 = (150 + 273,15)K
= 293,15K = 423,15K
Sea volumen inicial V1 P2 = 600mmHg
P1 = ¿? Entonces el V2 = 2V1
2º Utilizando la ecuación (14) para calcular la presión inicial:
P1V1 PV T1 V2
 2 2 ….(14) despejando P1  P2 x x remplazando valores
T1 T2 T2 V1
tenemos:
293,15 K 2V 1
P1  600 mmHgx x entonces P1 = 831,3 mmHg ó 831,3 torr
423,15 K V1

Respuesta: La presión inicial es 831,3 mmHg ó 831,3 torr


¿Resuelva este problema por un segundo método?

PROBLEMA 12: Un cilindro contiene 24kg de cloro líquido. ¿Calcular el volumen


en (m3) que el cloro ocuparía si fuera puesto al estado gaseoso a condiciones normales
de presión y temperatura (CNPT)?
Solución del problema: 1º Las propiedades de P,V,T del gas cloro a CNPT:
El cloro gas es diatómico Cl2, asumiendo comportamiento ideal.
P = 1 atm V = ¿? 1mol Cl2 = 22,414L = 71,0 g
T = 0ºC + 273 = 273,15K m = 24kg = 24 000g
Masa atómico del cloro es 35,5u entonces la masa molar del cloro es:
M = 71,0g/mol.
AMADO DE DIOS (amado_dedios@hotmail.com)[Escriba texto] Paá gina 22
Autor: Amado Crisógono Castro Chonta 2015-2
2º Utilizando la ecuación (21) para calcular el volumen del cloro en el estado gaseoso:
m m RT
PV  RT ……(21) despejando el volumen tenemos: V  x
M M P
1000 g L.atm
24kg ( )(0,08205 )(273,15K )
1kg mo lg .K
V  simplificando el volumen es
g
(71,0 ) (1atm)
mo lg

1m 3
V=7 576L transformando el volumen de L a m3 tenemos: V  7576 L( )
1000 L

V = 7,576 m3 Respuesta el cloro ocuparía un volumen de 7,576 m3

¿Resuelva este problema por un segundo método?

2, 1, 3b: DENSIDAD Y DENSIDAD MOLAR DE LOS GASES:


Densidad es el cociente indicado de la masa (m) de una sustancia por su volumen(V):
m  masa
Sea   densidad que se expresa por  ……………(32)
V  volumen

g g kg kg lb
Las unidades de la densidad son:   3
   3  ...(32,1)
cm mL L m pie3

La densidad molar es el cociente indicado de la densidad de una sustancia por su masa


molar:
 m m1
m  densidad molar ……(33) entonces al desarrollar m   
M MV  M  V

Obtenemos que la densidad molar ecuación (33) equivale a la ecuación (34) que es el
cociente indicado del número de moles (n) de una sustancia por su volumen (V):
n  númerodemoles
m  ………...(34) entonces sus unidades son:
V  volumen

 n mo lg mo lg mo lg mo lg
m  =     3 ………(34,1)
M V cm3 ml l m

2, 1, 3c: DENSIDAD RELATIVA DE LOS GASES IDEALES:


Es la relación existente entre la densidad de un gas y la densidad de un gas de
referencia; en este caso es generalmente el aire a las condiciones normales de presión y

temperatura (CNPT): aireCN = 1,289g/l = 1,289kg/m3 …………..(32,2)


Cuando se comparan dos densidades se deben hacer a las mismas condiciones de
presión y temperatura:

AMADO DE DIOS (amado_dedios@hotmail.com)[Escriba texto] Paá gina 23


Autor: Amado Crisógono Castro Chonta 2015-2
A (  A ) P .T
 A( B )  entonces r  ………(35)
B (  aire ) P.T

Donde: r es densidad relativa; A es la densidad del gas problema; aire es la


densidad del aire y B es la densidad del gas de referencia.
Aplicando la ecuación (32) a las sustancias “A” y “B”:
(  A ) P1T  m A / V A y (  B ) P1T  m B / V B ………………(32,3)
Dividiendo las ecuaciones (32,3) y aplicando la ley de AVOGADRO: “Los gases ideales
a las mismas condiciones de P y T a igual número de moles (nA= nB) sus volúmenes
son iguales (VA = VB ) ………..(36)
m A / VA m
r  entonces  simplificando r  A ………………(35,1)
mB / VB mB

Conocemos que la masa de una sustancia es: mA = nAMA y mB=nBMB reemplazando


en la ecuación (35,1) se obtiene:
nA M A MA
( r ) P , T  y simplificando si VA = VB  nA = nB entonces ( r ) P ,T  ….
nB M B MB
(37)
La densidad relativa de un gas ideal a las mismas condiciones de presión y temperatura
es una simple relación de sus masas molares.

PROBLEMA 13: ¿Hallar la densidad del gas: a) metano (CH4); b) etano (CH3-
CH3) ; c) O2; y d) etino; en g/l a condiciones normales de presión y temperatura
(CNPT)?
Solución del problema 13a: 1º Las propiedades del metano a CNPT son:
Asumiendo que el gas metano (CH4) se comporta como un GI a CNPT
P = 1 atm Vm = 22,414L/mol T = 273,15K M = 16g/molg
2º Utilizando la ecuación (23) para calcular la densidad del gas metano:
(1atm)(16 g / mo lg)
PM 
 ……(23) reemplazando valores L.atm
RT (0,08205 )(273,15 K )
mo lg .K

Respuesta: La densidad del gas metano a CNPT es = 0,714 g/L

PROBLEMA 14: Se tiene encerrado en un cilindro gas anhídrido sulfuroso (SO2)


Calcular: a) Su densidad (g/l) a 27ºC y 2 atm y

AMADO DE DIOS (amado_dedios@hotmail.com)[Escriba texto] Paá gina 24


Autor: Amado Crisógono Castro Chonta 2015-2
b) Su densidad relativa en el Sistema Internacional (SI) y en el Sistema de
Ingeniería Americano (SIA)
Solución del problema: 1º Las propiedades de P,V,T del SO2 son:
L.atm
T = (27 + 273,15)K P = 2 atm R = 0,082 05 mo lg .K

T = 300,15K MSO2 = 64,0 g/molg Maire = 28,9 g/molg


2º Utilizando la ecuación (23) para calcular la densidad del SO2:
PM
 …(23) reemplazando valores
RT

(2atm)(64,0 g / mo lg)

L.atm
(0,08205 )(300,15K )
mo lg .K

Respuesta “a”: La densidad del SO2 es:  = 5,200 g/L


3º Utilizando la ecuación (37) para calcular la densidad relativa del SO2:
MA 64,0 g / mo lg
r  …..(37) reemplazando valores su r = 28,9 g / mo lg
MB

Respuesta “b” su densidad relativa es r = 2,21


La densidad relativa es una propiedad termodinámica del estado de una
sustancia y es adimensional.

PROBLEMA 15: La masa molar de un gas “A” es la tercera parte de aquel de otro gas
“B”. A igual V y T la densidad del gas “A” es el doble de aquella del gas “B”. ¿Si dos
bulbos idénticos contienen separadamente a los gases “A” y “B”, calcular la relación de
la presión del gas A con respecto a la del gas B?

&&&&&&&&&&&YO TE AMO MUCHO&&&&&&&&&&&&&


“Mi amor nunca se apartará de tu lado” Isaías 54,10
“Yo nunca te dejaré, ni te abandonaré” Hebreos 13,5
“No tengas miedo, porque Yo estoy contigo” Isaías 43,5
“No tengas miedo, solamente confía en mi” San Marcos 5,36

AMADO DE DIOS (amado_dedios@hotmail.com)[Escriba texto] Paá gina 25


Autor: Amado Crisógono Castro Chonta 2015-2
“Dame hijo mío tu corazón, ámame, y que tus ojos hallen deleite en mis
caminos” Proverbios 23,26
“Tú eres precioso a mis ojos, de gran estima y Yo te amo mucho” Isaías
43,4
&&&&&&&&&&&&&&&&&&&&&&&&&&&&&&&&&&&&

AMADO DE DIOS (amado_dedios@hotmail.com)[Escriba texto] Paá gina 26


Autor: Amado Crisógono Castro Chonta 2015-2
2, 1, 4 MEZCLAS DE GASES IDEALES:
2, 1, 4a: LEY DE LAS PRESIONES PARCIALES DE DALTON:

Diagrama Nº 11: De las presiones parciales de Dalton.

La presión total (P) de una mezcla de gases es igual a la suma de las presiones parciales
(Pi) de los componentes de una mezcla gaseosa confinados en un mismo volumen
(V=cte.) y a una misma temperatura (T=cte.) en todo el proceso termodinámico.
Sea P = Presión total y Pi es la presión parcial del componente “i” de la mezcla;
entonces la presión total de una mezcla gaseosa es:
m

( P   Pi  Pa  Pb  Pc  ...  Pm ) V,T…....(38)
i a

AMADO DE DIOS (amado_dedios@hotmail.com)[Escriba texto] Paá gina 27


Autor: Amado Crisógono Castro Chonta 2015-2
Se llama presión parcial (Pi) de cada gas en una mezcla gaseosa a la presión que
dicho gas ejercerá si ocupará solo el volumen total de la mezcla y a la misma
temperatura:
Pa V = na RT ecuación de estado del gas (a)…. (19a)
Se tiene 3 gases Pb V = nb RT ecuación de estado del gas (b)…. (19b)
Pc V = nc RT Ecuación de estado del gas (c)…. (19c)
Sumando las tres ecuaciones tenemos: (Pa + Pb + Pc) V = (na + nb + nc) RT ……(39)
Si la presión total de la mezcla es: P = Pa + Pb + Pc………..(38,1)
Y también el número de moles totales de la mezcla es: n = na+nb+nc…(40)
Entonces reemplazando las ecuaciones (38,1) y (40) en la ecuación (39) obtenemos la

ecuación (41): PV = nRT….(41) Esta ecuación es idéntico a la ecuación (19)


por lo tanto la misma ecuación se puede utilizar tanto para mezcla de gases ideales y
gases ideales puros.
Ahora dividiendo cada ecuación (19a), (19b) y (19c) entre la ecuación (41) se obtiene:

Pa  na  n
    Pa = xaP donde a  xa se llama fracción molar del componente “a”
P  n n
de la mezcla gaseosa igual a su fracción de presión, y para el componente “b” será Pb =
xbP donde xb es fracción molar del componente “b” de la mezcla igual a su fracción de
presión, y para el componente “c” será Pc = xcP donde xc es la fracción molar del
componente “c” de la mezcla igual a su fracción de presión. En general entonces la
fracción molar del componente “i” de la mezcla es igual a su fracción de presión; y la

presión parcial de un componente “i” de la mezcla es pi = xi P………(42)


La fracción de presión parcial ejercida por cada componente de una mezcla gaseosa es
directamente proporcional a su fracción molar (xi), llamada también concentración

pi ni
molar, en dicha mezcla:  xi = …..(42a) y el porcentaje de su concentración
p n
molar de un componente de la mezcla gaseosa es equivalente a su porcentaje de presión:

 pi  ni
 100  xi (100) = (100)………… (42b)
 p n

AMADO DE DIOS (amado_dedios@hotmail.com)[Escriba texto] Paá gina 28


Autor: Amado Crisógono Castro Chonta 2015-2
Dividiendo la ecuación (39) entre la ecuación (41) y simplificando se obtiene:

p a  p b  p c n a  nb  nc
 = 1 …………..(43) que nos dice que la sumatoria de las
p n

fracciones de presión de todos los componentes de una mezcla es igual a la unidad:


p a pb pc p a  pb  pc p
    = 1…….(43a)
p p p p p

Y la sumatoria de las fracciones molares de todos los componentes de una mezcla es = 1


n a n b n c n a  nb  n c n
     1 …………(43b)
n n n n n

John Dalton (Eaglesfield, Cumberland (Reino Unido), 6 de septiembre de 1766 -


Manchester, 27 de julio de 1844), fue un naturalista, químico y matemático,
meteorólogo británico. En 1801 enunció la ley de las presiones parciales y la de las
proporciones múltiples.3

En 1808 expuso la teoría atómica en la que se basa la ciencia física moderna. Demuestra
que la materia se compone de partículas indivisibles llamadas átomos. También ideó una
escala de símbolos químicos, que serán luego reemplazadas por la escala de Berzelius.4

En 1826 se le concedió la Medalla de Oro de la Royal Society de Londres, así como de


la Academia Francesa de las Ciencias.

Falleció en Manchester en 1844, a la edad de 78 años. Más de 40.000 personas


acudieron al funeral para presentar sus respetos al científico5 6

2, 1, 4b: LEY DE LOS VOLUMENES PARCIALES DE AMAGAT:

Diagrama 12: Ley de los volumenes parciales de Amagat

AMADO DE DIOS (amado_dedios@hotmail.com)[Escriba texto] Paá gina 29


Autor: Amado Crisógono Castro Chonta 2015-2
El volumen total (V) ocupado por una mezcla gaseosa es igual a la suma de los
volúmenes parciales (Vi) de sus gases componentes que se mantienen a temperatura
constante (T=cte.) y presión constante (P=cte.) en todo el proceso termodinámico.
El volumen parcial de un componente “i” de una mezcla gaseosa (Vi), es el volumen
que ocupará aquel componente gaseoso si el solo estuviera presente a la misma presión
y temperatura que tiene la mezcla:
Va , Vb, Vc son los volúmenes parciales de los gases componentes de una mezcla a
las mismas condiciones de P y T entonces el volumen total de una mezcla de gases
ideales es: ( V = Va + Vb + Vc )P,T ………(44)
Se tiene tres gases ideales que forman una mezcla:
Va P = na RT ecuación de estado del gas ideal (a)…….(19d)
Vb P = nb RT ecuación de estado del gas ideal (b)…….(19e)
Vc P = nc RT ecuación de estado del gas ideal (c)…….(19f)
Sumando las tres ecuaciones se obtiene: (Va + Vb + Vc) P = (na + nb + nc) RT ….(45) Si
el volumen total de la mezcla es: V = Va + Vb + Vc …………………….(44a)
Y también el número de moles totales de la mezcla es: n = na + nb + nc ……(40)
Entonces reemplazando las ecuaciones (44a) y (40) en la ecuación (45) obtenemos la

ecuación: PV = nRT…(41). Esta ecuación es idéntico a la ecuación (19)


por lo tanto se puede utilizar tanto para mezcla de gases ideales y gases ideales puros.
Ahora dividiendo cada ecuación (19d), (19e) y (19f) entre la ecuación (41) se obtiene
Va n a
las ecuaciones: za =  = ya donde la fracción volumétrica del componente “a”
V n

Vb nb
de la mezcla es igual a su fracción molar; entonces V a = yaV; zb =  = yb; donde
V n
la fracción volumétrica del componente “b” de la mezcla es igual a su fracción molar;

Vc nc
entonces Vb = ybV; y zc =  = yc donde la fracción volumétrica del componente
V n
“c” de la mezcla es igual a su fracción molar; entonces Vc = ycV.
En general el volumen parcial de un componente “i” de la mezcla es igual al producto

de su fracción molar por el volumen total de la mezcla: Vi = yi V…(46) y


La fracción del volumen parcial (zi) ejercida por cada componente de una mezcla
gaseosa es directamente proporcional a su fracción molar (y i), o concentración molar,

AMADO DE DIOS (amado_dedios@hotmail.com)[Escriba texto] Paá gina 30


Autor: Amado Crisógono Castro Chonta 2015-2
Vi ni
en dicha mezcla: zi =  yi = …..(47) y el porcentaje de la concentración molar
V n
de un componente de la mezcla gaseosa es equivalente al porcentaje de su fracción

V  ni
volumétrica: 100zi=  i 100  yi (100) = (100) entonces % zi  % yi …………
V  n

(47a)

PROBLEMA 16: ¿Calcular la composición molar porcentual del oxigeno y del


nitrógeno en el aire?
1º La composición volumétrica aproximada del aire es: Oxígeno 21% y nitrógeno 79%
en volumen.
2º Hacer una Tabla para calcular composición molar
Componentes Volumen % volumen = % molar
21
O2 21L x100  21% 21%
100
79
N2 79L x100  79% 79%
100
(aire) 100 L Aplicando la ley de Amagat: %zi = %yi
Respuesta: La composición molar porcentual del oxígeno es 21% y del nitrógeno79%.

PROBLEMA 17: La composición volumétrica aproximada del aire seco es O2=21,0% y


N2 = 79,0%. ¿Hallar su composición en masa y masa molar de la mezcla?
1º Para determinar la composición volumétrica, %zi = 100(Vi/V), de una mezcla:
Gas Vi %zi = 100(Vi/V)
Mezcla gaseosa a Va 100(Va/V)
b Vb 100(Vb/V)
c Vc 100(Vc/V)
Aire V = V a + Vb + Vc 100
2º Para determinar la composición molar ,%yi = 100(ni/n), de una mezcla:
Gas ni %xi = 100(mi/m)

a na 100(na/n)

Mezcla gaseosa b nb 100(nb/n)

c nc 100(nc/n)

Aire n = na + nb + nc 100

AMADO DE DIOS (amado_dedios@hotmail.com)[Escriba texto] Paá gina 31


Autor: Amado Crisógono Castro Chonta 2015-2

3º Para determinar la composición en masa, %xi = 100(mi/m), de una mezcla:


Gas mi 100 yi = %yi
a ma 100 (ma/m)
Mezcla gaseosa b mb 100 (mb/m)
c mc 100 (mc/n)
Aire m = m a + mb + mc 100
4º Tomando como base de cálculo 100 moles de aire:
Gas %yi=%zi yi ni(mol) Mi mi %xi= 100(mi/m)
O2 21 0,21 21 mol 32g/mol 672g 672
x100  23,3
2884
2212
N2 79 0,79 79 mol 28g/mol 2 212g x100  76,7
2884
Aire 100% n=100 mol m=2884g 100%
 Para calcular el número de moles del oxígeno se multiplica las 100 mol de aire
por su porcentaje molar n02 = xO2(100) = (0,21)100 mol entonces n02 = 21
mol O2
 Para calcular la masa del oxígeno se multiplica su número de moles por su masa
molar: mO2= ni. Mi = 21 mol (32g/mol) entonces la masa es mO2 = 672g O2
 Para calcular la masa total de la mezcla se suman la masa del oxígeno con la
masa del nitrógeno: m = mO2 + mN2 = 672g + 2 212g entonces la masa total
es m = 2 884g
 Para calcular la composición en masa del oxígeno se multiplica por 100 la
fracción másica del oxígeno:
%xO2 = 100(mO2/m) = 100(672g/2 212g) entonces la composición en masa del
oxígeno es: %xO2 = 23,3 % Respuesta
 Calculando la masa molar de la mezcla (M) se divide la masa total de la mezcla
(m) entre el número de moles totales de la mezcla (n):M = m/n=2 884g/100mol
entonces la masa molar de la mezcla es: M = 28,84g/mol :Respuesta

PROBLEMA 18: En la fabricación del ácido sulfúrico a partir de la tostación de


sulfuros metálicos, el gas de tostación sale a 155ºC y 550 mmHg y contiene 8,1% en
volumen de SO2. ¿Calcular la masa en gramos de SO2 contenida por m3 de dicho gas de
tostación?

AMADO DE DIOS (amado_dedios@hotmail.com)[Escriba texto] Paá gina 32


Autor: Amado Crisógono Castro Chonta 2015-2
Solución del problema: 1º Las propiedades de P,V,T del gas de tostación en la salida
son: Sea el volumen del gas de tostación 1m3 = 1000 L
Asumir que el SO2 se comporta como un GI
T = (155 + 273,15)K = 428,15K P = 550 mmHg
%yi = 8,1%  porcentaje en volumen de SO2 MSO2 = 64,0g/molg
2º Calcular el volumen en litros del SO2 en el gas de tostación:
VSO2 = 1000L(0, 081) = 81 L de SO2
3º Para calcular la masa en granos de SO2: se aplica la ecuación (21,2)
PVM
m …..(21,2) reemplazando valores
RT
(550 mmHg )(81L)(64,0 g / mo lg)
m
 L.atm  760 mmHg
 0,08205 (428,15º K )( )
 mo lg .K  1atm

m = 106,8 g de SO2

Respuesta: Hay 107 gramos de SO2 por cada m3 de gas de tostación.


Otro método: 1º Es calcular el volumen del SO2 a CNPT sea (Va), usando la ecuación
P1V1 PV P  Ta 
 a a …..(14) despejando Va  V1  1  
T1 Ta  Pa  T1 
 550mmHg  273,15 K 
Reemplazando valores Va  81L  
 760mmHg  428,15 K 
Entonces el volumen del SO2 a CNPT es Va = 37,397L
2º Calcular la masa del SO2 por cada m3 del gas de tostación:
A CNPT: 1 molg SO2 = 64,0g = 22,414 L
(37,397 L)(64,0 g )
m 37, 397L entonces m 
( 22,414 L)

m = 106,8 g de SO2
Se demuestra que por los dos métodos se llega al mismo valor de 106,8 g de SO 2
por cada m3 del gas de tostación.
Para resolver un problema hay muchos caminos, entonces tenemos que
buscar el camino más corto y simple.

PROBLEMA 19: Un hidrocarburo tiene una densidad al estado de vapor igual a 2,55
g/L a 100ºC y 760mmHg. El análisis indica que esta sustancia contiene un átomo de

AMADO DE DIOS (amado_dedios@hotmail.com)[Escriba texto] Paá gina 33


Autor: Amado Crisógono Castro Chonta 2015-2
hidrógeno por cada átomo de carbono. ¿Cuál es la fórmula molecular de este
compuesto?
Solución del problema: 1º Los datos del problema son:
Densidad del vapor  = 2,55 g/L Temperatura T = (100 + 273,15)K = 373,15K
Presión P = 760 mmHg = 1 atm. Relación carbono-hidrógeno CnHn y su
masa molar empírica es ME = 13,0 g/mol
2º Calcular la masa molar del compuesto: Se usará la ecuación (21,3)
mRT
M  …..(21,3); introduciendo el parámetro densidad y despejando masa
VP

RT
molar M  ….(21,4); reemplazando valores:
P
L.atm
( 2,55 g / L)(0,08205 )(373,15 K )
mo lg .K entonces la masa molar
M 
1atm
verdadera es: M = 78, g/molg
3º Calcular la fórmula molécula del hidrocarburo:
Se divide la fórmula molar verdadera con respecto a la fórmula empírica para obtener
cuántas (n) veces la fórmula verdadera contiene a la fórmula empírica:

78 g / mo lg
13 g / mo lg
 6CH . Luego la fórmula verdadera es 6CH = C6H6 El benceno.

Respuesta: El hidrocarburo es el benceno C6H6.

PROBLEMA 20: Un bulbo de vidrio de una capacidad de 200ml, al llenarlo con


cierto gas a 25ºC y 740mmHg tiene una masa de 27,611g. Al reducir la presencia del
gas a 150mmHg, la masa del bulbo con este resto de gas fue de 27,411g. ¿Calcular la
masa molar del gas y la masa del bulbo de vidrio?
SOLUCIÓN: 1º Los datos del problema son:
V = 200mL T = (25 + 273,15)K = 298,15K P1 = 740 mmHg P2 = 150 mmHg
La masa del estado inicial: m1 = 27,611g y la masa del estado final: m2 = 27,411g
M, V y T son comunes para los dos estados M = ¿?
2º Calcular la masa molar de la sustancia en función de la cantidad de gas desalojada:

m1 m2
P1V  RT …….Estado 1 P2V  RT …….Estado 2
M M
Restando las dos ecuaciones para eliminar la masa del bulbo de vidrio tenemos:

AMADO DE DIOS (amado_dedios@hotmail.com)[Escriba texto] Paá gina 34


Autor: Amado Crisógono Castro Chonta 2015-2
RT RT
V(P1 – P2) = (m1 – m2) ...(21,6) entonces M = (m 1 – m2) …
M V(P1 - P2)

(21,7)
Calculando la presión del gas desalojado:
P = P1 – P2 = (740 – 150)mmHg = 590mmHg
Calculando la masa del gas desalojado m=(m1 – m2)=(27,611 – 27,411)g = 0,200g
Reemplazando valores en la ecuación (21,7) tenemos:
 L.atm 
(0,200 g ) 0,08205 (298,15º K )
 mo lg .K 
M 
 
; La masa molar es M =
1atm
 590mmHg ( ) (0,2l )
 760mmHg 

31,5g/ml
3º Calcular la masa del gas en el estado inicial: Usar la ecuación

P1VM
mG1  ……….(21,2) …………Reemplazando valores tenemos:
RT

(740mmHg )(0,2 L )(31,5 g / mo lg)


mG1 
 L.atm  760mmHg
 0,08205 ( 298,15º K )( )
 mo lg .K  1atm

La masa del gas en el estado inicial es: mG1 = 0,251 g

4º Calcular la masa del bulbo de vidrio: mbv = m1 – mG1 = (27,611 – 0,251) g


Respuesta: La masa del bulbo de vidrio es mbv = 27,36 g

PROBLEMA 21: Por electrolisis de una salmuera se obtiene en el ánodo una mezcla
de gases que tiene la siguiente composición en masa:
Cl2 = 70,0% Br2 = 25,0% O2 = 5,0%
¿Hallar la:
a) Composición de la mezcla gaseosa en volumen?
b) Densidad de la mezcla en g/L a 270ºC y 1 atm?
c) Densidad relativa (o gravedad específica) de la mezcla a CNPT?
d) Masa molar de la mezcla?
e) Composición molar de la mezcla gaseosa?

Solución del problema: 1º Los datos el problema son:


AMADO DE DIOS (amado_dedios@hotmail.com)[Escriba texto] Paá gina 35
Autor: Amado Crisógono Castro Chonta 2015-2
Composición en masa de la mezcla gaseosa es:
Cl2 = 71,0% Br2 = 24,0% O2 = 5,0%
Temperatura de cada componente y de la mezcla es: T = (270 + 273,15K = 543,15K
La presión de la mezcla es: 1 atm.  = ¿? r = ¿?
2º Tomar como base de cálculo 100g de mezcla gaseosa:
3º Hacer una tabla para calcular las composiciones de la mezcla:
Gas %xi xi mi(g) Mi ni %y i = %zi
Cl2 71,0 0,710 71,0g 71g/mol 1,00molg 76,34
Br2 24,0 0,240 24,0g 160g/mol 0,15molg 11,45
O2 5,0 0,050 5,0g 32g/mol 0,16mol 12,21
Mezcla 100% 1,0 m=100g n=1,31molg 100%
 Para calcular la masa del cloro se multiplica 100g de mezcla por su fracción
másica mCl2 = yCl2(100g) = (0,71)100g entonces m0Cl2 = 71g Cl2
 Para calcular las moles de cloro se divide la masa del cloro entre su masa molar:
nCl2= mCl2/MCl2 = 71,0g/ (71g/mol) entonces el número de moles es n Cl2 =
1,00molg Cl2
 Para calcular las moles totales de la mezcla se suman las moles de los
componentes de la mezcla: n = nCl2 + nBr2 + nO2= 1,00molg + 0,15molg +
0,16molg entonces las moles totales de la mezcla es n = 1,31molg

 Para calcular la composición molar del cloro se divide las moles de cloro por las
moles totales de la mezcla y se multiplica por 100: %yi = 100( nCL2/ n)

reemplazando valores tenemos: %yi=100(1,00molg/1,31molg) = 76,34%

Respuesta (e).
 Para calcular la composición volumétrica: Por la ley de Amagat la composición
molar de un componente de la mezcla es igual a su composición volumétrica:
entonces para el gas cloro tenemos %yCl2 = % zCl2 =76,34% :Respuesta (a)
 Calculando la masa molar de la mezcla (M) se divide la masa total de la mezcla
(m) entre las moles totales de la mezcla (n):M = m/n=100g/1,31molg entonces
la masa molar de la mezcla es: M = 76,34g/mol :Respuesta (d)
 Calcular la densidad de la mezcla en g/l: aplicando la ecuación la ecuación (23)
(100 g / 1,31mo lg)(1atm)
MP m 
 …(23); reemplazando valores  L.atm 
RT  0,08205  543,15 K 
 mo lg .K 

AMADO DE DIOS (amado_dedios@hotmail.com)[Escriba texto] Paá gina 36


Autor: Amado Crisógono Castro Chonta 2015-2
La densidad de la mezcla es:  = 1,713 g/L
 Para calcular la densidad relativa de la mezcla gaseosa se debe transformar la

CNPT T1
densidad de la mezcla a CNPT y aplicando las ecuaciones: ( 
1 TCNPT

 543,15 K 
)m,P...(11 )reemplazando valores CNPT  (1,713g / l )  entonces la
 273,15 K 

densidad de la mezcla a CNPT es: CNPT = 3,406g/l


(  A ) P .T
Y para calcular la densidad relativa se usa la ecuación:  r  ……(35)
(  aire ) P.T
3,406 g / L
Reemplazando valores r == 1,289 g / l entonces rRespuesta

(c)
PROBLEMA 22: Un bulbo con llave y sin aire, se llena con gas CO 2 a la presión
del ambiente y por diferencial de pesada se encuentra que contiene 4,40g del gas a
una determinada temperatura de T(K). Luego se coloca el bulbo en un baño de
temperatura constante, que marca una temperatura de 30ºC más alta que la primera.
Allí, estando la salida de la llave fuera del agua, se abre ésta hasta que la presión del
gas CO2 retoma al valor original. Entonces el bulbo contiene 3,96g de CO2 ¿Calcular
el valor de la temperatura T(K) inicial?
Solución del problema: 1º Los datos del problema en los estados inicial y final
son: Asumiendo que el CO2 se comporta como un GI
ESTADO INICIAL ESTADO FINAL
m1 = 4,40g CO2 m2 = 3.96g CO2
V = cte. V = cte
T1 = T(K) T2 = (T + 30)K
P1 = P MCO2 = 44 g/molg P2= P1 = P
m1 m2
PV  RT PV  R(T  30)
M M
2º Calcular el valor de la temperatura:
 Restando las dos ecuaciones de estado: m1T = m2 (T + 30)
 Desarrollando y despejando la temperatura: m1T = m2 T + (m2) 30
30m 2
(m1 – m2) T = 30m2  T
m1  m 2

AMADO DE DIOS (amado_dedios@hotmail.com)[Escriba texto] Paá gina 37


Autor: Amado Crisógono Castro Chonta 2015-2
30 K (3,96 g )
 Reemplazando valores la temperatura inicial es: T  T= 270K
( 4,40  3,96) g

Respuesta la temperatura inicial es 270K


PROBLEMA 23: A) ¿Calcular la presión total ejercida por una mezcla gaseosa que
se compone de 320g de O2, 1120g de N2 y 20g de H2; Si dicha mezcla está contenida en
un cilindro de 1m3 de capacidad a la temperatura de 17ºC? B) ¿Calcular la fracción
molar y presión parcial de cada componente?
Solución del problema: 1º Datos del problema:
V = 1m3 = 1000 L T = 17 + 273,15 = 290,15K
mO2 = 320g mN2 = 1120g mH2 = 20g
2º Calculo de las moles totales de la mezcla: n = nO2 + nN2 + nH2
1mo lg 1mo lg
nO2 = 320 gx  10mo lg O2 nN2 = 1120 gx 28 g  40mo lg N2
32 g
1mo lg
nH2 = 20 g ( )  10mo lg H 2 n = 10 + 40 + 10 = 60molg
2g
3º Calcular la presión total de la mezcla: Aplicar la ecuación (19)……..PV = nRT
� L.atm �
0,08205
� (290,15 K )

� mo lg.K � entonces la presión es P = 1,429 atm
P  (60mo lg)
1000 L
4º Calcular la fracción molar y presión parcial de cada componente:
Respuesta: Composición molar Presión parcial de cada componente:
10
X O2  x100  16, 67% PO2 = XO2 (1,429 atm) = 0,238atm
60
40
XN2  x100  66, 66% PN2 = XN2 (1,429 atm) = 0,953atm
60
10
XH2  x100  16, 67% PH2 = XH2 (1,429 atm) = 0,238atm
60
100,00% Presión total de la mezcla 1,429atm

PROBLEMA 24: En un tubo de vidrio lleno de mercurio, invertido sobre una cuba
de Hg, se ha recogido por desplazamiento de este, que llenaba el tubo un volumen de
700cc CO2 en las condiciones siguientes: Temperatura 30ºC, presión atmosférica
740mmHg y la altura sobre el nivel del Hg dentro del tubo fue de 14cm sobre el nivel
del mercurio en la cuba. ¿Calcular a) La presión del gas b) y la masa
del gas recogido?

AMADO DE DIOS (amado_dedios@hotmail.com)[Escriba texto] Paá gina 38


Autor: Amado Crisógono Castro Chonta 2015-2
Solución del problema: 1º Datos del problema:
T = (30 +273,15)K = 303,15K Ptotal = Patm = 740 mmHg Pman = 100mmHg
2º Calcular la presión del gas; Pgas = Ptotal – Pman = (740 – 140)mmHg = 600mmHg
3º Calcular el volumen el gas a CNPT: aplicando la ecuación (14)
V1 P1 V P V PT 2
 2 2 = cte. Entonces V2 = 1 1 (14) reemplazando valores
T1 T2 T 1P 2
700mL(6OOmmHg )(273,15)
 V 2 entonces V2 = 498 mL = 0,498L de CO2
(760mmHg )(303,15K )
4º Calcular la masa del gas CO2 recogido:
�1mo lg � �44 g �
mCO2  0, 498 LCO 2 � �
� � entonces mCO2 = 0,978g
�22, 414 L �
�1mo lg �

Respuesta: la presión del CO2 es 600mmHg y su masa es 0,978g


PROBLEMA 25: Se quiere preparar una mezcla gaseosa que contenga 10% molar de
butano (C4H10) y 90% molar de neón. En un cilindro sin aire se introduce butano
gaseoso hasta que su presión es de 1 atm. Luego se inyecta dentro del cilindro gas neón
comprimido, para formar una mezcla de la composición deseada. El volumen del
cilindro es de 20 L y la operación se realiza a 25ºC. ¿Calcular:
a) Los moles de butano presente? Rpta: 0,82 moles
b) Los moles de neón necesario? Rpta: 7,38 moles
c) La presión total de la mezcla gaseosa final? Rpta: 10 atm.
d) Composición másica y volumétrica?
Otros datos: Masa atómico del: neón 20,2u; hidrógeno 1,01u; carbono 12,01u
PROBLEMA 26: Un gas consiste de una mezcla de etano y butano. Un balón de
200ml se llena con este gas a la presión de 750torr y a 20ºC. Por diferencia de masa la
masa del gas es de 0,384 6g. ¿Calcular las composiciones: a) molar; b) volumétrico; y c)
masa de la mezcla?
PROBLEMA 27: Un cilindro de 10 L de capacidad contiene un gas a 27ºC y 5,00
atm. El gas se escapa a razón de 20ml por minuto, medidas a CNPT. ¿Calcular la
presión del gas que queda en el cilindro al cabo de 10 horas y la masa molar de la
mezcla, asumiendo que la temperatura permanece constante a 27ºC?
PROBLEMA 28: Un frasco de 22L de capacidad contiene 40,0g de gas argón, y una
masa de gas hidrógeno, medidos a una determinada presión y temperatura. La densidad
de la mezcla gaseosa es de 2,00g/L. La masa atómica del argón es 40,0u. ¿Calcular:

AMADO DE DIOS (amado_dedios@hotmail.com)[Escriba texto] Paá gina 39


Autor: Amado Crisógono Castro Chonta 2015-2
a) Los gramos de hidrógeno presentes en el frasco? Rpta 40g de hidrógeno.
b) La masa molar promedio de la mezcla gaseosa? Rpta. 14,7 g/mol
PROBLEMA 29: A 220ºC y 474 mmHg, una masa de 1,388g de cierta sustancia
orgánica, ocupa un volumen de 420mL. El análisis del gas da una composición
elemental en masa siguiente: C: 70,60%; H 5,88% y O…el resto. ¿Calcular la masa
molar de la sustancia y su fórmula molecular?
PROBLEMA 30: El óxido de plata se descompone completamente a temperaturas
superiores a 600K, transformándose en plata sólida y oxígeno gaseoso:
2Ag2O(s)  4Ag(s) + O2(g)
Cierta masa de óxido de plata da, al descomponerse 83,8 mL de oxígeno recogido sobre
agua a 25ºC y 756 mmHg. La presión de vapor del agua a 25ºC es 23,8 mmHg.
¿Calcular la masa de oxígeno y de óxido de plata en la muestra?
PROBLEMA 31: Cierta cantidad de carbonato de calcio se descompone
completamente por calentamiento produciendo CaO(s) y desprendiendo 2,5 L de gas
CO2 que se recoge en agua medidos a 40ºC y 755,3 mmHg. La presión de vapor del
agua a 40ºC es 55,3mmHg.

2, 1, 5 DETERMINACION DE MASAS MOLECULARES DE LAS


SUSTANCIAS DE BAJA TEMPERATURA DE VAPORIZACIÓN:

Diagrama Nº 13: El agua se transforma en vapor a 100ºC y 1 atm de presión


 Existen diferentes métodos para determinar las masas moleculares de los vapores
generalmente de sustancias sólidos o líquidos a temperaturas ordinarias.
 Los más conocidos son:
- Método de J.B.A. DUMAS (1826)
- Método de VICTOR MEYER (1878)

AMADO DE DIOS (amado_dedios@hotmail.com)[Escriba texto] Paá gina 40


Autor: Amado Crisógono Castro Chonta 2015-2
 Método de J.B.A. DUMAS: En un matraz de vidrio, de masa conocida, de unos 250
mL de capacidad volumétrica, que presenta un tubo estrecho de salida que se puede
cerrar a la lámpara o con una llave, se introduce la muestra problema. Este aparato se
calienta en un baño de temperatura constante, a unos 20ºC por encima del punto de
ebullición de la sustancia problema. Se continúa calentando hasta que todo el líquido
se ha convertido en vapor y se ha expulsado completamente el aire del matraz. El
tubo estrecho del matraz se cierra luego a la lámpara o con la llave, se saca del baño
caliente, se deja enfriar y se pesa. Como se conoce la masa del matraz se puede
calcular la masa del vapor que lo llena, efectuando una corrección para el aire
contenido en el matraz vacío. Como se conocen todos los parámetros: el volumen
del matraz, la temperatura del baño y la presión atmosférica se puede calcular la
masa molecular de la sustancia problema.
El Método de Dumas se puede utilizar para determinar la masa molecular de
sustancias que tiene temperaturas altas de vaporización, y se emplean globos de
porcelana y platino.
 Método de VICTOR MEYER Consiste en colocar en una ampolla de vidrio una
cantidad determinada de líquido, se le evapora a temperatura constante, originando
que sea desalojado del aparato un volumen equivalente de aire, que es medida en una
bureta de gases, a temperatura y presión conocidas. El volumen del vapor se mide
indirectamente que es igual al volumen de aire desalojado por aquel.

PROBLEMA 32: Por el método de Víctor Meyer se determinó la masa molecular de


una sustancia orgánica desconocida. Después de la evaporación de 0,152 5g de la
muestra líquida, se midió la expulsión de 35,05mL de aire de la bureta de gases; la
presión barométrica fue de 730mmHg y la temperatura de 20ºC. El análisis químico dio
la composición en masa de la sustancia: C: 22,10%; H: 4.58%; Br: 73.32%.
Calcular: a) la masa molecular de la sustancia: b) y averiguar su fórmula molecular.
Solución del problema: 1º Los datos del problema son:
Vmuestra = Vaire desplazado = 35,05mL = 0,035 05L
P = 730mmHg T = (20 + 273,15)K = 293,15K m = 0,152 5g
2º Calcular la masa molecular de la sustancia: se aplica la ecuación (21,1)

AMADO DE DIOS (amado_dedios@hotmail.com)[Escriba texto] Paá gina 41


Autor: Amado Crisógono Castro Chonta 2015-2
� L.atm �
(0,1525 g ) �
0, 08205 (293,15 K )

mRT � mo lg.K �
M  ...(21,1) � M 
VP � 1atm �
(0, 03505L )(730mmHg ) � �
�760mmHg �
M = 109 g/molg; entonces Rpta. (a): La masa molecular de la sustancia es 109g/molg
3º Determinar la fórmula empírica y su masa molecular de la sustancia problema:
Elemento %m mi(g) Mi(g/molg) ni  atm
C 22,10 24,09 12,0 2 molg 2
H 4,58 5,00 1,0 5 molg 5
Br 73,32 79,9 79,9 1 molg 1
100,00 M =109.00g
 Tomando como base 109g de sustancia problema, para determinar la masa de
carbono se multiplica la masa molar de la sustancia por el % en masa del
carbono: mc = M.%zc = 109g(0,221) entonces la mc = 24,09g.
 Para determinar el número de mol de átomos se divide la masa de carbono por
su masa molecular: #atm = mc / Mc = 24,09g/(12,0 g/molg) entonces #atm =
2.
Respuesta (b): La fórmula molecular de la sustancia problema será: C2 H5 Br

PROBLEMA 33: Por el método de Dumas se determinó el peso molecular de una


sustancia problema (líquido a la temperatura ambiente). Unos 10 mL de esta sustancia
(CCl4) se colocó en un bulbo de Dumas de material de vidrio terminando en un capilar,
Se calentó luego en un baño de temperatura constante (ejemplo. H 2O en ebullición)
hasta que todo el líquido se vaporizo, expulsando el aire del bulbo. Se leyó la lectura del
barómetro del laboratorio, cerrándose inmediatamente el capilar del bulbo. En el
momento del cierre del bulbo estaba lleno de vapor a la presión barométrica que es 1
atm a la temperatura del baño de 100ºC. Entonces “T” y “P” son conocidos, y la masa
de la sustancia problema (m) se calcula por diferencia de las pesadas del bulbo cerrado
y del bulbo vacío, sin vapor aunque con aire; el volumen “V” se determina llenando el
bulbo con agua y pesándolo. En una experiencia de laboratorio se determinaron los
siguientes datos:
ma = bulbo + agua = 411g
mb = bulbo + aire = 51,43g
mc = bulbo + CC14 (vapor) = 52,86g
Solución del problema: 1º Los datos del problema son:

AMADO DE DIOS (amado_dedios@hotmail.com)[Escriba texto] Paá gina 42


Autor: Amado Crisógono Castro Chonta 2015-2
ma = 411g mb = 51,43 g mc = 52,86g P = 1atm
T = (100 + 273,15)K = 373,15K
2º La capacidad volumétrica del bulbo es V= (ma-mb)/H2O = (411 – 51)g (1mL/g)=
360mL. Entonces el volumen del bulbo es 360 mL.
3º Calcular la masa de aire en el estado inicial, en el momento de pesar el bulbo (mc): la
masa de aire a las condiciones de laboratorio es 1 atm y 18ºC, se usa la ecuación (21,2)
PVM
m aire  …..(21,2) reemplazando valores
RT
(1atm)(0,36 L)(28,96 g / mo lg)
aire 
m � L.atm �
�0, 08205 �(291,15º K )
� mo lg.K �

maire = 0,44 g de aire


4º Calcular la masa del bulbo vacío es mbv = mb – maire = (51,43 – 0,44)g = 50,99g

5º Para determinar la masa de la muestra será: mCC14vapor = mc – mbv = (52,86-50,99)g =

1,87g entonces la masa es: mCC14(vapor) = 1,87g.

6º Calcular la masa molar de la sustancia: se aplica la ecuación (21,1)


� L.atm �
(1,87 g ) �
0, 08205 �(373,15 K )
mRT � mo lg.K �
M  ...(21,1) � M 
VP (0,360 L)(1atm)

M = 159 g/molg; entonces Rpta. (a): La masa molar del CCl4 es 159g/molg
El valor real es 154g/molg.

2, 1, 6: LEY DE GRAHAM DE LA DIFUSIÓN

AMADO DE DIOS (amado_dedios@hotmail.com)[Escriba texto] Paá gina 43


Autor: Amado Crisógono Castro Chonta 2015-2
Diagrama 14: Ley de Graham

La difusión es el proceso por el cual una sustancia se distribuye uniformemente en el


espacio que la encierra o en el medio aprovechable para ella. Ejemplo1: Si se conecta
dos tanques conteniendo el mismo gas a diferentes presiones, en corto tiempo la presión
es igual en ambos tanques.
Ejemplo2: Si se introduce una pequeña cantidad de gas en un extremo de un tanque
cerrado que contiene otro gas B, rápidamente el gas A se distribuya uniformemente
por todo el tanque.
La difusión es una consecuencia del movimiento continúo y elástico de las moléculas
gaseosas.
El producto de la presión (P) de un GI por su volumen (V) es igual a los dos tercios de

2
la energía cinética (Ec) o fuerza viva de sus moléculas PVm  Ec  RT …….(48)
3
1 1
Si se toma como base una molg de GI: E c  Mv 2  PVm  Mv 2  RT …..(49)
2 3
3RT PVm
Entonces la velocidad cuadrática media de un GI es:  
2
3 …....(50)
M M
Luego anunciamos los postulados siguientes:

AMADO DE DIOS (amado_dedios@hotmail.com)[Escriba texto] Paá gina 44


Autor: Amado Crisógono Castro Chonta 2015-2
1. Las velocidades de las moléculas de un gas aumenta igualmente con el aumento

3RT 3PV
de temperatura   …..…..(50,1)
M M
2. La energía cinética de las moléculas de un gas ideal es directamente

3
proporcional a la temperatura absoluta: E c  RT ….(51)
2
3. Y la energía cinética (Ec) promedio de las moléculas de todos los gases son
idénticas a la misma temperatura constante, ecuación (51); luego la velocidad
cuadrática media de las moléculas de un GI es:
3P
v2  3P 3RT 3PV
�M �entonces v    ……(52)
� � p M M
�V �
La velocidad de difusión (v) es directamente proporcional a la velocidad de las
moléculas de dicho gas (v), de acuerdo con la ley de GRAHAM (1829)

v = Kv   K  PT  V
………..(53)
1
v
A las mismas condiciones de P y T de dos gases diferentes: A y B; se puede calcular la
relación entre sus velocidades de difusión: va, vb, va y vb

3RT 3RT
Para el gas A: va = Kva = K y para el gas B: vb= Kvb  K
Ma Mb

Dividiendo tenemos: donde: t es tiempo


3RT
K
 
Va
Vb
PT
1

Ma
3RT
  
Va
Vb
PT
1

M b tb

M a ta
…..(53,1)
K
Mb

3P 3P
Para el gas A: va = Kva  K ; y para el gas B: vb = KVb  K …(53,2)
a b

3P
K
Tenemos:  Va
Vb
PT
1

a
3P

V
( Va 
b
b
a
)P,T…….(53,3)
K
b

AMADO DE DIOS (amado_dedios@hotmail.com)[Escriba texto] Paá gina 45


Autor: Amado Crisógono Castro Chonta 2015-2

La velocidad de difusión de un gas es inversamente proporcional a la raíz cuadrada de


su respectiva masa molar o de su densidad cuando la presión del gas y su temperatura se
mantienen constantes.
Cuando la presión del gas y su temperatura es el mismo la velocidad de difusión del gas
es inversamente proporcional al tiempo para una misma longitud de recorrido. Ejemplo
cuanto más pesada es la molécula mayor es el tiempo de recorrido para una longitud
dada debido a que su velocidad de difusión es menor.
La Efusión es la velocidad de escape a través de un pequeño orificio, de las
moléculas de un gas contenidos en un recipiente.
PROBLEMA 30: A condiciones normales de presión y temperatura (CNPT)
calcular la velocidad cuadrática media de los siguientes gases: a) hidrógeno, b)
nitrógeno, c) oxígeno, d) amoniaco, e) cloruro de hidrógeno, f) cloroformo, g) y metano.
Solución a: 1º Datos del problema: Masa molecular del hidrógeno = 2,016g/molg
Volumen molar = 22,414L/molg T = 273,15K P = 1atm = 1,013x 106dina/cm2
R = 8,314x107erg/(molg.K)
2º Calcular la velocidad cuadrática media del hidrógeno, aplicar la ecuación (52)

3PV
  ……………(52) reemplazando valores
M

3(1, 013x106dina / cm2)(22, 414 x103cm3)


  [(1g .cm / s 2) / 1dina ]
(2, 016 g / mo lg)

183 814cm = 1838 m


Respuesta: La velocidad cuadrática media del hidrógeno es 1838 m

PROBLEMA 31: Calcular la rapidez de efusión relativa del: a) He y del O2; b) H2 y


He; c) H2 y O2; y d) He y Ne.
Solución a: 1º Datos del problema: La masa molar del O2 y del He son, MO2 =
32,00g/nolg y MHe = 4,00g/molg.
2º Calcular la rapidez de efusión relativa del He y del O2: Asumir que los dos gases
están a las mismas condiciones de presión y de temperatura y luego aplicar la

ecuación  
VHe
VO 2
PT
1

MO 2
MHe
………….(53,1); reemplazando valores

AMADO DE DIOS (amado_dedios@hotmail.com)[Escriba texto] Paá gina 46


Autor: Amado Crisógono Castro Chonta 2015-2

 
VHe
VO 2 PT
1

32, 00 g / mo lg
4, 00 g / mo lg
Entonces  
Va
Vb
PT
1
 2,83. Respuesta:

La rapidez de efusión relativa del He y del O2 es de 2,83

PROBLEMA 32: Calcular la temperatura a la cual la velocidad cuadrática media de


la molécula nitógeno es de: a) 1000m/s, b) 800m/s, c) 2000m/s, d) y 4000m/s.
PROBLEMA 33: Un volumen de nitrógeno pasa en 24s por el orificio de un
efusiómetro y bajo las mismas condiciones de temperatura y presión un volumen
identico de una mezcla gaseosa de oxígeno y dióxido de carbono se difunden en 30s.
¿Calcular la composición volumétrica del CO2?
PROBLEMA 34: ¿Calcular la velocidad relativa de difusión de los gases del UF6
preparados a partir de los isótopos del uranio: U-235 y U-238. Sugerencias: usar 5 cifras
significativas en los calculos?
PROBLEMA 35: El tiempo de escape de un gas diatómico a través de un pequeño
orificio es de 24,4 minutos y el correspondiente tiempo del hidrógeno es de 5,5 min.
¿Cuál es la masa molar del gas diatómico?
PROBLEMA 36: Una mezcla en volumen de 6 partes de neón y una parte de argón
es difundida a través de un pequeño orificio hacia un espacio evacuado. ¿Cuál es la
composición de la mezcla que pasa primero?

II) GASES REALES

Diagrama N° 15: Las curvas de los Gases Reales


Muchos gases se comportan como gases ideales a temperaturas y presiones ambientales,
es decir a presiones bajas y temperaturas altas. Si embargo la mayoría de los gases

AMADO DE DIOS (amado_dedios@hotmail.com)[Escriba texto] Paá gina 47


Autor: Amado Crisógono Castro Chonta 2015-2
sujetos a condiciones de alta presión, se apartan de la ley de los gases ideales; y ésta
proporciona una descripción cada vez más pobre de la conducta del gas.

La necesidad de describir y expresar matemáticamente el comportamiento de un gas, y


que concuerden con los datos experimentales nos permite señalar que en esencia,
existen cuatro métodos que pueden aplicarse en los cálculos para pronosticar el
comportamiento de los gases reales; ellos son:
A) Las ecuaciones de estado
B) Las gráficas del factor de compresibilidad
C) Las propiedades calculadas
D) Los datos experimentales reales

Las leyes de los gases no pueden aplicarse al estado líquido.

2,2 LAS ECUACIONES DE ESTADO:

Diagrama N° 16: Estados Termodinámicos


Relacionan las propiedades de P.V.T. de una sustancia pura (o de mezclas) mediante
correlaciones teóricas o empíricas. Existen más de un centenar de ecuaciones de
estado, de tal manera que éstas deben predecir los estados termodinámicos
experimentales con exactitud razonable en toda la fase gaseosa.

A continuación se indican algunas de las ecuaciones de estado, para una mol de gas:
a
a) Van de Waals: ( P  2 ) (V - b) = RT
V
Donde V es volumen molar, P es presente, R es constante universal de los gases
ideales, T es temperatura, a , b y c,  y  son constante, cuyos valores dependen
de las propiedades de los gases que se estudia.
RT a
b) Clausius: P = -
V  b T (V  c )
2

RT a
c) Lorentez: P = 2 (V +b) -
V V2

RT a
d) Berthelot: P = -
V  b TV 2

AMADO DE DIOS (amado_dedios@hotmail.com)[Escriba texto] Paá gina 48


Autor: Amado Crisógono Castro Chonta 2015-2

RT A
e) Keyes: P = - ;
V S (V  1) 2

donde: S = e - /V

f) Ecuación virial de estado.- Es una ecuación de la forma:


PV B (T ) C (T )
=1+ + + …….
RT V V2

2,2,1 Ecuación de estado de Van der Waals VDW): Diagrama 17:

Es la primera ecuación de estado propuesto, por Van der Waals, y se uso, para
describir el comportamiento de un gas real. Es a la vez la ecuación de estado
más simple.

Van Der Waals introdujo en la ley del gas ideal los siguientes conceptos:
 El efecto de las fuerzas de atracción existente entre las moléculas, es decir
cuando una molécula alcanza la pared del recipiente que lo contiene, choca
con menor intensidad, debido a que las moléculas adyacentes ejercen fuerzas
de atracción, lo que es lo mismo disminuye la presión ejercida contra la
pared o presión del gas. Entonces la presión corregida del gas será:
a
(P + )
V2
Donde (a) es una constante dimensional que depende de la naturaleza de las
fuerzas atractivas del gas.

AMADO DE DIOS (amado_dedios@hotmail.com)[Escriba texto] Paá gina 49


Autor: Amado Crisógono Castro Chonta 2015-2
 El efecto de volumen, se debe a que las moléculas de un gas real tiene un
volumen libre disponible o volumen compresible que es la diferencia entre el
volumen total molar (V) y el volumen propio de las moléculas (b). La
constante (b) de VDW es en realidad cuatro veces el volumen real de las
moléculas, que esta determinada por la mínima distancia que dos moléculas
pueden acercarse. Entonces el volumen corregido del gas será: (V - b)
La ecuación de Van de Waals será:
a
Para una mol (P + ) (V - b) = RT ……. (1)
V2
 2a
Para n moles (P + ) (V - b) = RT ……. (2)
V2

Las constantes a y b se determinan aplicando la ecuación de VDW a los


datos experimentales P, V, T y particularmente a los valores en el punto
crítico, entonces la ecuación de VDW es una expresión semiempírica. En la
Tabla # 1 se puede encontrar los valores de las constantes a y b de VDW de
algunos gases. En caso de que no se encontraran, se tiene que calcular
teniendo en cuenta que la isoterma crítica pasa por el punto de inflexión en el
punto crítico y que la pendiente vale cero en ese punto, (@P / @V) T = 0,
igualmente la segunda derivada, (@2p / @V2) = 0, por lo tanto para la
ecuación de estado de VDW (1), ordenando tenemos:
RT a
P= - ……. (3)
V b V 2
Tomando la primera y segunda derivada a la ecuación (54a) tenemos:
 P  RT 2a
  =- 2 + ……………. (4)
 V  T (V  b) V3

 2R  2 RT 6a
  =-
2  3 + ……. (5)
 V  T (V  b) V4

Debido a que las dos derivadas son iguales a cero en el punto crítico (Tc, Pc, Vc)
tenemos:
RTc 2a a RTc
-
(Vc  b) 2
+ 3 = 0  3 = 2(Vc  b ) 2 ……. (6)
Vc Vc

AMADO DE DIOS (amado_dedios@hotmail.com)[Escriba texto] Paá gina 50


Autor: Amado Crisógono Castro Chonta 2015-2
2 RTc 6a a RTcVc
-
(Vc  b) 3
+ 4 = 0  3 = ……. (7)
Vc Vc 3(Vc  b)3

RTc a
Pc = - …..… (8)
(Vc  b) Vc 2

Los primeros miembros de las ecuaciones (6) y (7) son iguales, luego:
RTc RTcVc
2 =  Simplificando se obtiene que:
2(Vc  b) 3(Vc  b) 3

V c = 3b ……. (9)
Y al reemplazar en (6) y simplificando se obtiene que:
a = (9/8) RTc Vc ……. (10)
Luego reemplazando (9) y (10) en (8), tenemos:
Pc = (3/8) RTc/ Vc ……. (11)
Combinando las ecuaciones (8), (9), (10) y (11) se demuestran que:
Pc = a/ (27b2) ……………. (12)
8a
Tc = ……………. (13)
( 27 Rb)

27 R 2Tc 2
a= ……………. (14)
64 Pc
RTc
b= ……. (15)
8 Pc
Las unidades de las constantes a y b son:
a < ---- > atm (cm3/molg)2 ; psia (pie3/mol/b)2
Pa (m3/molg)2
b < ---- > cm3/molg; pie3/mol lb; m3/molg

La ecuación de VDW puede resolverse fácilmente para la presión (o para la


temperatura):
RT a
Para una mol : P = - 2 ……. (3) o
V b V

RT  2a
Para n moles: P = V  b - ……. (3a)
V2

AMADO DE DIOS (amado_dedios@hotmail.com)[Escriba texto] Paá gina 51


Autor: Amado Crisógono Castro Chonta 2015-2
En caso de resolver el volumen (v) o moles (n) la ecuación (2a) se convierte en
una relación de tipo cúbico:
RT  2a  3 ab
V3 - (b + )V2+( )V- =0 ……. (2b)
P P P

En consecuencia para determinar el volumen o moles se tendría que:


a) Utilizar el método de aproximaciones sucesivas.
b) Obtener la gráfica de L ecuación suponiendo diferentes valores de V (o de n),
y observar a que valor de V (o de n) en la curva se logra la intersección con
la abscisa.
c) Emplear el método de NEWTON.

TA B LA 1
Constantes a y b de la ecuación de VDW y constantes críticas de algunos gases:
a b Tc Pc Vc
GAS M atm (cm /molg)3 ºK atm cm3/molg
(L/molg)2
Aire 28,970 1,328 36,62 132,6 37,20 92,59
Ar 39,948 1,345 32,20 151,2 48,00 74,94
Cl2 70,910 6,495 56,22 417,2 76,10 124,00
CH4 16,040 2,255 42,78 190,7 45,80 98,78
C2H2 26,040 4,368 51,23 309,5 61,60 113,12
C2H4 28,050 4,476 57,16 283,1 50,50 124,32
C2H6 30,070 5,490 63,80 305,4 48,20 147,96
C3H8 44,098 8,668 84,42 369,9 42,00 200,98
C4H10 58,120 14,312 121,15 425,2 37,50 254,96
C6H6 78,110 18,000 115,20 562,6 48,60 260,05
C6H5CH3 92,142 24,062 146,35 594,2 41,60 315,53
C6H5Cl 112,560 24,433 145,33 632,2 44,60 308,46
C6H5Br 157,010 28,581 153,93 670,2 44,60 323,22
CH3OH 32,040 9,525 67,05 513,4 78,50 117,78
C2H5OH 46,070 12,023 84,08 516,2 62,99 167,32
CH3Cl 50,490 7,471 64,86 416,5 65,80 143,43
CHCl3 119,390 15,170 10,22 536,2 54,20 238,37
CCl4 153,840 20,380 138,34 556,4 45,00 276,20

AMADO DE DIOS (amado_dedios@hotmail.com)[Escriba texto] Paá gina 52


Autor: Amado Crisógono Castro Chonta 2015-2
CO 28,010 1,485 39,83 133,2 34,56 93,20
CO2 44,010 3,592 42,78 304,2 72,88 94,23
CS2 76,140 11,630 76,88 552,0 78,30 172,70
CH3COOH 60,050 17,630 10,72 594,2 57,35 172,30
(C2H5)2O 74,120 17,400 13,44 467,4 35,60 281,60
He 4,000 0,034 23,72 5,3 2,26 58,30
H2 2,016 0,244 26,62 33,3 12,80 65,40
HCl 36,470 3,668 40,82 324,6 81,53 87,30
HBr 80,917 4,453 44,22 363,3 84,00
H2O(g) 18,016 5,478 30,63 647,4 218,50 55,60
Ne 20,183 0,211 17,08 44,5 25,90 41,70
N2 28,020 1,347 38,62 126,2 33,50 90,20
NO 30,006 1,340 27,96 179,2 64,80 58,10
NO2 46,010 5,295 45,38 432,0 100,00 82,30
N2O 44,020 3,782 44,15 309,8 71,80 96,80
NH3 17,034 4,187 37,32 405,5 111,30 72,50
O2 32,000 1,362 31,88 154,4 49,70 74,20
SO2 64,063 6,718 56,52 430,7 77,80 122,20
Equivalencias de unidades para pasar de un sistema a otro sistema de unidades:
1,000 00 atm (L/molg)2 = 256,591 atm (pie3/mollb)2
= 106 atm (cm3/molg)2= 3,771 x 103 psia (pie3/mollb)2
1,000 dm3/molg = 103 cm3/mol = 16,018 pie3/mollb
1lb = 453,592 37g; 1kg = 1000g = 2,205 lb; 1TM = 1000kg = 2 205lb.
PROBLEMA 1: Calcular la presión ejercida por un mol de CO2 que ocupa el volumen
de 10,0 L a 100ºC por dos métodos:
a) La ecuación de los gases ideales y
b) La ecuación de VDW
SOLUCIÓN:
1. Base de cálculos y datos del problema
V = 10,0 dm3/molg T = 373,2ºK
 = 1,0 molg R = 0,082 057 dm3.atm/molg K
a = 3,60 atm (dm3/molg)2; b = 0,0428 dm3/molg (datos de la tabla 1)

2. Por el método de gases ideales la presión será:


P = RT/V  P = 0,082 057 (L atm/molg K)(373,2K)/(10,0L/molg)
P = 3,06 atm: Rpta (a)

3. Por el método de VDW, la presión será:


RT a
P= - 2
V b V

AMADO DE DIOS (amado_dedios@hotmail.com)[Escriba texto] Paá gina 53


Autor: Amado Crisógono Castro Chonta 2015-2

(0, 082 057 L atm / mol K )(373, 2 K ) 3,60 atm( L / mo lg) 2


P= -
(10, 0 L / mo lg  0, 0428 L / mo lg)
(10,0 L / mo lg) 2
P = 3,04atm: Rpta (b)

4. Discusión y Conclusión: Se observa que el gas, CO 2, a las condiciones dadas


se comporta como un gas ideal porque el resultado obtenido por los dos
métodos es prácticamente igual.

PROBLEMA 2: Calcular por dos métodos el volumen ocupado por una mol de O 2 a
-88ºC y 44,7 atm
1. Base de cálculo y datos del problema
 = 1,00 molg O2 ; T = 185,2K
P = 44,7 atm ; a = 1,36 atm (L/molg)2
b = 0,031 9 L/molg
2. Por el método de gases ideales: V = RT/P, reemplazando valores tenemos:
V = (0,082 057 L atm/molg K) (185,2K/44,7 atm)
V = 0,34 L/molg: Respuesta
3. Aplicando la ecuación de VDW:
a
(P + ) (V – b) – RT = 0
V
 Resolviendo por el método de aproximaciones sucesivas y gráfico
1ro. Reemplazar los valores en la ecuación (1)
1,36
(44,7 + ) (V – 0,031 9) – 0,082 06 (185,2) = 0
V
2do. Hacer una tabla, dar valores a V y calcular 
1,36
V (44,7 + ) (V – 0,0319) – 15,197 5 
V2

0,34 (56,4647) (0,3081) – 15,197 5 2,201


0,25 (66,46) (0,2181) – 15,197 5 -0,701

El valor que se busca (V) esta entre 0,34 cercano a 0,25 L/molg; para encontrarlo
graficar  vs V

AMADO DE DIOS (amado_dedios@hotmail.com)[Escriba texto] Paá gina 54


Autor: Amado Crisógono Castro Chonta 2015-2

0,27

-1.0

Diagrama 18: Método Gráfico

Respuesta: En este caso el valor será 0,27 1/molg que corresponde al intersepto con
la abscisa.

2,2,2 FACTORES DE COMPRESIBILIDAD


Se discutirá la ecuación de estado del factor de compresibildiad (Z) y los métodos
para su cálculo. En los primeros experimentos se encontró que en el punto crítico
todas las sustancias están aproximadamente en el mismo estado de dispersión
molecular, entonces será conveniente considerar que las propiedades
termodinámicas y físicas deberían ser similares. Estas observaciones originó la ley
de los estados correspondientes, que expresa que en el estado crítico todas las
sustancias deben comportarse en forma semejante.

El estado crítico (Pc, Tc, Vc), es el conjunto de condiciones físicas en las cuales la
densidad y otras propiedades del líquido y del vapor son idénticas. En particular
solo para el caso de un componente puro, es la máxima temperatura a la cual puede
existir en equilibrio el líquido-vapor. Los valores experimentales de la Temperatura
crítica (Tc) y Presión crítica (Pc) para sustancias puras pueden obtenerse de la Tabla
1 en caso que no se encuentre consultar los textos de Reid y Sherwood (5), capítulo
2, y el manual del Ingeniero Químico (6), pags. 3-142, 3-143, 3-287, 3-288, 3-289;
en las cuales se describe y se analizan los métodos para determinar las constantes
críticas de diversos compuestos y elementos.
Diagrama 19: P vs V de una sustancia pura de la región líquido-vapor

AMADO DE DIOS (amado_dedios@hotmail.com)[Escriba texto] Paá gina 55


Autor: Amado Crisógono Castro Chonta 2015-2

Diagrama 20: P vs T de una sustancia pura y diagrama de fases

Al correlacionar las propiedades de estado (P, V, T) de los gases con su estado


crítico (Pc, Tc, Vc), se observó que todas las sustancias se comportan en forma
similar en su estado reducido (Pr, Tr, Vr); particularmente cualquier sustancia tiene
el mismo volumen reducido (Vr) a la misma temperatura (Tr) y presión reducida
(Pr) ver Figura 1. Las propiedades reducidas se calculan de la siguiente manera:
V T P
Vri = …… (16); Tr = …… (17) y Pr = …… (18)
Vci Tc Pc
Matemáticamente se puede expresar que la ecuación funcional en el estado reducido es
f(Pr,Tr,Vr)=0 …………… (19)
FIGURA 1: Comprobación Experimental de la relación PR /TR = cte/ VR para un
conjunto de gases (metano, etano, propano, pentano, heptano y etileno).
VR=0.80

VR=1.00

AMADO DE DIOS (amado_dedios@hotmail.com)[Escriba texto] Paá gina 56


Autor: Amado Crisógono Castro Chonta 2015-2
VR=0.60 VR=1.50

VR=2.00
PR

3.00

4.00

TR

2,2,2a ECUACIÓN DE ESTADO DEL FACTOR DECOMPRESIBILIDAD


Se expresa como: PV = ZRT …………… (20) o
PV= ZRT …… (21)
Es una modificación de la ecuación de estado de un gas ideal, que describe el
comportamiento (P, V, T) de los gases reales a lo largo de una variedad mucho más
amplia de condiciones. Ha sido desarrollada para relacionar los conceptos de la ley de
los estados correspondientes y la ley del gas ideal. La cantidad adimensional (Z) se
denomina factor de compresibilidad. Un valor de Z = 1.000 corresponde a un
comportamiento de un gas ideal.

ESTIMACIÓN
El factor de compresibilidad de un gas depende de la temperatura y de la presión del
gas, Z = f(T, P). Z varía de un gas a otro para un mismo valor de T y P. ejemplo: el valor
de Z para N2 a 200ºK y 100 bar es 0,845 5, mientras que el valor de Z para el CH 4 a las
mismas condiciones de P y T es de 0,365 7

AMADO DE DIOS (amado_dedios@hotmail.com)[Escriba texto] Paá gina 57


Autor: Amado Crisógono Castro Chonta 2015-2

Fig. 2 Fig. 3
Factor de compresibilidad como Compresibilidad como una
función de la temperatura y la función de la temperatura y
presión presión reducidas

En la Biblioteca del Ingeniero Químico (6) pags 3-143 a 3-147, incluyen valores de Z =
f(T, P) para el aire, Argón, CO2, CO, H2, CH4, N2, O2, y Vapor de agua

PROBLEMA 3: Se almacenan 388 kg de H2 en un tanque a 150ºK y 70 atm de presión


absoluta. Estimar el volumen del tanque utilizando la ecuación de estado del factor de
compresibilidad.
SOLUCIÓN:
a. Base de cálculos: 388 kg de H2
b. El número de molkg de H2 será:

 1molkg H 2 
nH2 = 388 kg  
 2,016 kg 
388
nH2 = 2,016 molkg H2

c. El valor de Z para H2 será 1,050 7 obtenido de la bibliografia (6) (pág 3-

145): T = 150ºK y P = 70 atm  Z = 1,0507

d. Usar la ecuación PV = ZRT para calcular el volumen 


V = ZnRT/P, reemplazando valores tenemos:

 388  �150 K ��0, 082057m3 atm �


V = 1,0507  molkg  � �� �
 2,016  �70 atm � K molkg

AMADO DE DIOS (amado_dedios@hotmail.com)[Escriba texto] Paá gina 58


Autor: Amado Crisógono Castro Chonta 2015-2
V = 35,56 m3

Sin embargo al construir las gráficas del factor de compresibilidad con respecto a los
estados reducidos Pr y Tr, Z = z (Tr, Pr), se observa que para todos los gases los valores
de Z a la misma Tr y Pr coinciden aproximadamente en un mismo punto, ver figura Nº

3. Entonces para estimar Z=z (Tr, Pr) se usarán las Gráficas 4 al 8 del factor de
compresibilidad generalizado, que fueron preparados por Nelson y Obert (7). El
procedimiento a seguir para estimar Z es el siguiente:
1. Buscar o estimar valores de Tc y Pc
2. Si el gas es H2 ó He, corregir los valores críticos:
(Tc) corregido = Tc + 8 ºK …… (22)

(Pc) corregido = Pc + 8 atm …… (23)

3. Calcular los valores reducidos de las variables conocidas, mediante las


fórmulas:
Tr = T/Tc …… (17)

Pr = P/Pc …… (18)

Vri = V/Vci …… (24)

Vci = RTc/Pc …… (25)

Donde: Vri es volumen molar reducido ideal y Vci es volumen molar crítico
ideal. Las constantes reducidas deben ser adimensionales.
4. Una vez calculadas dos de las propiedades reducidas usar la gráfica del
factor de compresibilidad, que ha sido seleccionado para estimar Z.

PROBLEMA 4: Un cilindro de 5,00 pie3 contiene 50,00 lb de propano (C3H8) y se


encuentra en un lugar, donde recibe directamente el calor de un caldero. El manómetro
indica que la presión es de 665 psig. ¿Cuál es la temperatura del C 3H8 en el cilindro,
calcular por 3 métodos?
a) Gases ideales
b) Ecuación de estado de VDW y
c) Ecuación de estado del factor de compresibilidad (Z).

SOLUCIÓN:

AMADO DE DIOS (amado_dedios@hotmail.com)[Escriba texto] Paá gina 59


Autor: Amado Crisógono Castro Chonta 2015-2
1. Base de cálculos: 50,00 lb de C3H8
2. Por el métodos de gases ideales, la temperatura del gas será:
T = PV/nR reemplazando valores tenemos:

(679, 7 psia ) (5, 00 pies 3 ) 44,098 lb


T= psia pie3 ( )
(50, 0 lb) (10, 73 )
mol lb R 1 mol lb
T = 279,3R: Rpta (a)

3. Por el método de gases reales, ecuación de estado de VDW


 2 a V  b
Ecuación T = (P + ) ( R ) reemplazando valores tenemos:
V2
n = 50,0 lb (1molb/44,098 lb) entonces n = (50,0/44,098) mollb
 50,000 
 50,00 
2 5,0    1,35
  3,27 x10 4  44,098 
T = (679,7 +  44,098  )( )
 50,000 
(5,0) 2    10,73
 44,098 
T = 673,3R: Rpta (b)
4. Por el método del factor comprensibilidad, Z:
a) Construir la siguiente tabla:
Pc Tc Vci Pr Vri V Z
(atm) (ºK) (L/molg) (L)

42,0 369,9 0,7226 1,10 0,381 0,275 0,405


b) Calcular Vci = RTc/Pc
L atm
(0, 082057 ) (369,9 K )
Vci = mol K , luego Vci = 0,722 7 L/molg
42, 0 atm

c) Calcular Vri = V/Vci reemplazando valores tenemos:


5, 0 pie3 28,31685 L 2, 205 mol lb
( ) ( 3
) ( )
Vri = 50, 00 / 44, 098) mollb 1, 00 pie 103 mo lg
0, 7227 L / mo lg

Vri = 0,381

d) Calcular Pr = P/Pc, reemplazando valores tenemos:

AMADO DE DIOS (amado_dedios@hotmail.com)[Escriba texto] Paá gina 60


Autor: Amado Crisógono Castro Chonta 2015-2

 679,7 psia   1,00 atm 


Pr = 
 42,0 atm   14,696 psia 
  
Pr = 1,10

e) Calcular Z usando el diagrama de Z:  Z = 0,405

1
f) Calcular T = (PV/nR)
Z
Reemplazando valores tenemos:

T = 279,3 R/0,405

T = 689,6 R: Rpta (c)

La respuesta más confiable es T= 689,6R por que se calculó por el

método del factor de comprensibilidad (Z)

PROBLEMA 5: Un reactor de amoniaco gaseoso se llenó con este reactivo


equivalente a 8,0 lb. La presión en el reactor es de 150 lbf/pulg 2 abs y la temperatura es
de 250ºF. Determinar el volumen del reactor utilizando tres métodos:
a) Gases ideales
b) Ecuación de VDW
c) Factor de compresibilidad
SOLUCIÓN:

1. Base de cálculos: 8,00 lb de NH3


P = 150 psia T = 709,67 R M = 17,034 lb/mollb

n= (8,00/17,034) mollb  n = 0,469 65 mollb 

R = 10,73 (psia pie3/mollbR)

2. Por el método de gases ideales, el volumen del gas será:

V = nRT/P

(0, 469 65 mol lb) (10, 73 psia. pie3 / mollbR) 709, 67 R


V=
150 psia

V = 23,84 pie3 : Rpta (a)

AMADO DE DIOS (amado_dedios@hotmail.com)[Escriba texto] Paá gina 61


Autor: Amado Crisógono Castro Chonta 2015-2
3. Por el método de gases reales, utilizando la ecuación de estado de VDW.
 2a
(P + ) (V - b) – RT = 0 …… (2)
V2
a = 15 800 psia (pie3/mol lb)2
b = 0,597 5 pie3/mol lb
Reemplazando valores en la ecuación de VDW:

15 800(0,4700) 2
(150 + ) (V - 0,47000(0,5975) - 0,47 (10,73) (709,67) =
V2
0
Construir una tabla dando valores a la incognita (V)

3490,22
V (150 + )(V - 0,281) - 3578,9 
V2
Construyendo factores y sumandos:

1º 2º 3º 4º
V (pie3)
24,0 156,060 23,719 3578,94 122,65
23,0 156,600 22,719 3578,94 -21,2
23,2 156,485 22,919 3578,94 7,5
Construir una gráfica con los puntos calculados (V,  ) para determinar el
volumen, de tal manera que el V = 23,15 pie3 cuando  = 0,000 entonces el
V = 23,15 pie3 Respuesta…… (b)

Diagrama 21: Método Gráfico de vs V

AMADO DE DIOS (amado_dedios@hotmail.com)[Escriba texto] Paá gina 62


Autor: Amado Crisógono Castro Chonta 2015-2

7.5
23,15

23,2

-21,2

4. Por el método del factor de compresibilidad (Z)

a) Construir la siguiente tabla:

Tc Pc P T Tr Pr Z
(K) (atm) psia (R)

405,5 111,3 150 709,67 0,9723 0,091 7 0,963

b) Calcular Tr = T/Tc
Tr =(709,67R/405,5K) (1,0K/1,8R) entonces Tr = 0,972 3
c) Calcular Pr = P/Pc
Pr = (150 psia/111,3 atm) (1,00 atm/14,696 psia) luego Pr = 0,091 7
d) Calcular Z usando las gráficas del factor de compresibilidad
Z = 0,963
e) Calcular V = ZnRT
V = 0,963 (23,84 pie3)  V = 22,96 pie3
El volumen del reactor es 23,0 pie3 Rpta (c)
La respuesta más confiable es V = 23 pie3 calculado por el método del factor
de comprensibilidad (Z).
PROBLEMA 6: Calcular el valor de Z en el estado crítico usando las gráficas del

factor de compresibilidad.

SOLUCIÓN:

1. En el punto crítico los valores de Tr y Pr serán:


Pr = Pc/Pc  Pr = 1,000
Tr = Tc/Tc  Tr = 1,000
2. Usar la gráfica # 8 para estimar ZTr y luego calcular (Z):

AMADO DE DIOS (amado_dedios@hotmail.com)[Escriba texto] Paá gina 63


Autor: Amado Crisógono Castro Chonta 2015-2
Z Tr = 0,27  Z = 0,27
El valor de Zc es 0,27

PROBLEMA7: Un tanque de oxígeno contiene 100,000 pie3 de 02 seco medidos a


400K y 10 bar. Se desea pasar este gas a un tanque cuya capacidad es 10 pie 3 y que se
encuentra a la temperatura de 220K ¿Cuál será la presión en el tanque; calcular usando
3 métodos?
SOLUCIÓN:
1. Base de cálculos y datos del problema:
Estado P V T
(1) 10 bar = 9,869 2atm 100 pie3 = 2831,7L 400K
(2) ? 10 pie3 = 283,17 L 220K
2. Por el método de gases ideales, la presión P2 será:
P2 = 10 bar (100 pie3/10pie3) (220K/400K)
P2 = 55 bar o P2 = 54,28 atm
Las molgO2 que se encuentran en el recipiente será: nO2 = PV/RT
nO2 = (9,869 2atm) (2831,7L)/(400K)(0,082 057 L atm/(mol.K))
nO2 = 851,44 molg
3. Por el método de la ecuación de VDW
Calcular el número de molg usando los datos del estado (1)
n2 a
3,1 Primero hacer una tabla: Ecuación: (P + ) (V - nb) - nRT = 0
V2
a = 1,36 atm (1/molg)2 b = 0,031 9 a/molg

n 2 (1,36)
(9,8692 + ) (2831,7 - 0,031 9n) - 0,082 057 (400) n= 0
2831, 7 2

n (9,8692 + 1,696 x 10-2n2) (2831,7-0,031 9n) -32,822 8n 

850 9,992 00 2804,6 27 899,4 124


860 9,994 64 2804,3 28 277,6 -199

n = 853,8 molg

3,2 Calcular P2 usando los datos del estado (2)


nRT n2a
P= -
V  nb V 2

AMADO DE DIOS (amado_dedios@hotmail.com)[Escriba texto] Paá gina 64


Autor: Amado Crisógono Castro Chonta 2015-2

853,8 (0,082 06) (220)


P2 = -

283,17  853,8 (0,031 9)

853,8 2 (1,36)
283,17 2

P2 = 47,86 atm P2 = 48,49 bar

4. Por el método de las gráficas del factor de compresibilidad:


Hacer una tabla para calcular Z y n con los datos del estado (1)
Pc Tc Pr Tr Z  = PV/ZRT
(atm) (ºK)
49,7 154,4 0,199 2,59 1,000 851,44 molg

4,2 Hacer una tabla para calcular Z y P2 com los datos del estado (2)
Pc Tc Pr Vri Tr Z P2
(atm) (ºK) 1/molg (bar)
49,7 154,4 0,3326 1,30 1,425 0,894 49,17

a) Calcular:

RTc 0,082 06(154,4)


Vci =  Vci
Pc 49,7
Vci = 0,255 L/molg

b) Calcular:

283,17 1
V=  V = 0,332 6 L/molg
851,44 mo lg

c) Calcular:

AMADO DE DIOS (amado_dedios@hotmail.com)[Escriba texto] Paá gina 65


Autor: Amado Crisógono Castro Chonta 2015-2

0,332 6 L / mo lg
Vri = V/Vci  Vri =
0,255 L / mo lg
Vri = 1,30
d) Calcular:
Tr = T/Tc  Tr = 220ºK/154,4ºK
Tr = 1,425
e) Calcular Z usando el gráfico de factor de compresibilidad  Z =
0,894
f) Calcular:

P2 = Z (RT/V)  P2 = 0,894 (55 bar)


P2 = 49,17 bar o P2 = 48,53 atm

2,3 MEZCLAS GASEOSAS DE LOS GASES REALES


Muchos de los problemas prácticos en la industria y en la mayoría de los casos
manejamos mezclas gaseosas, por lo tanto discutiremos diferentes métodos para
calcular las propiedades (P, V, T) de mezclas en base exclusivamente a las propiedades
de los componentes puros:
A) Ecuación de estado
B) Constantes promedios
C) Factor de compresibilidad medio (Zm)

2,3A ECUACION DE ESTADO


Cada una de las ecuaciones de estado para gases reales y puros se puede combinar
con las leyes de Dalton o las leyes de Amagat. Ejemplo las leyes de Dalton y
Amagat pueden combinarse con la ecuación de estado de VDW.

2,3A1) LA ECUACION DE ESTADO Y LA LEY DE DALTON


Usando la ecuación de estado de un gas real se calcula la presión parcial de
cada componente puro, para luego sumarles de acuerdo con la ley de Dalton y
obtener la presión total del sistema. Entonces cuando se usa la ecuación de
estado de VDW y la ley de Dalton tenemos las siguientes expresiones
matemáticas:

AMADO DE DIOS (amado_dedios@hotmail.com)[Escriba texto] Paá gina 66


Autor: Amado Crisógono Castro Chonta 2015-2
P = Pa + Pb + Pc + ....…… (26)
La presión parcial Pj para cada componente será:
 a RT 2
 a aa
Pa = - …… (27)
V   a ba V2

 b RT 2
 b ab
Pb = - …… (28)
V   b bb V2
Reemplazando las ecuaciones (27) y (28) en (26) tenemos la presión de la
mezcla: (Pt):
 a RT  b RT 1
Pt = RT ( + + ...) - (a2 aa + b2 ab + ...) ……
V   a ba V   b bb V2
(29)

2,3A2) LA ECUACION DE ESTADO Y LA LEY DE AMAGAT

Para calcular V(o ) de una mezcla; primero debe calcularse V (o ) de cada


componente y la suma de estos es el volumen total según la ley de Amagat. La
resolución de este problema es muy complicado. Para el caso particular de la
ecuación de estado de VDW y la ley de Amagat, tenemos las siguientes
expresiones matemáticas:
V = Va + Vb + Vc + ..... …… (30)

y luego estimar los valores de Va, Vb, Vc etc. (o a, b, c, etc.) usando la
ecuación de VDW. Para cada componente V (o n) es una ecuación cúbica
cuyo cálculo es complicada.

2,3B CONSTANTES PROMEDIO


Una forma simple y efectiva es usar las constantes promedio de la ecuación de
estado de un gas real, que permitirá resolver problemas de mezclas gaseosas. Para el
caso particular de la ecuación de estado de VDW se puede proceder como sigue:
bm = baYa + bbYb. + ..... …… (31)

a1/2m= a1/2aYa + a1/2Yb + ..... …… (32)

Las constantes promedio también pueden obtenerse a partir de las constantes


seudocríticas; entonces para la ecuación de VDW tenemos:

AMADO DE DIOS (amado_dedios@hotmail.com)[Escriba texto] Paá gina 67


Autor: Amado Crisógono Castro Chonta 2015-2

27 R 2Tc 2
a= …… (33) y
64 Pc 2
RTc
b= …… (34)
8 Pc

2,3C FACTOR DE COMPRESIBILIDAD MEDIO (Zm)


Para mezclas de gases se puede usar la ecuación de estado del factor de
compresibilidad:
PV = Zm (RT) …… (35)

Donde Zm es el factor de compresibilidad medio y se puede calcular usando las


siguientes expresiones:
Zm = ZaYa + ZbYb + ..... …… (36)

Sabemos que Z es una función de Pr y Tr; entonces es necesario decidir que presión
deberá usarse para calcular Pr.

2,3C.1) CONSIDERANDO LA LEY DE DALTON

Cada componente Zj se calcula a la Trj y a la presión parcial reducida de cada


componente gaseoso Prj. Esta se expresa como:
Prj = Pj/Pcj …… (37) y
Trj = T/Tcj …… (38)

2,3C2 DE ACUERDO CON LA LEY DE AMAGAT

Cada componente Zj se calcula a la Trj y P Rj total del sistema. Esta se expresa


como:
Prj = P/Pcj …… (39)
Trj = T/Tcj …… (40)

2,3D PROPIEDADES SEUDOCRÍTICAS


Conocido como el método de Kay. Los valores seudocríticos se calculan según las
expresiones siguientes:

AMADO DE DIOS (amado_dedios@hotmail.com)[Escriba texto] Paá gina 68


Autor: Amado Crisógono Castro Chonta 2015-2
Presión Seudocrítica:
P'c = PcaYa + PcbYb + ..... …… (41)

Temperatura Seudocrítica:
T'c = TcaYa + PcbYb + ..... …… (42)

Temperatura Seudocrítica ideal:


V'c1 = (Vci Y)a + (Vci Y)b + ..... …… (43)

RTc a RTc b
Luego (Vci)a = ; (Vci)b = …… (44)
Pc a Pcb

Los respectivos valores seudoreducidos son:


Presión Seudoreducida  P'R = P/P'c …… (45)
Temperatura Seudoreducida  T'r = T/T'c …… (46)
Volumen Seudoreducida  V'r = V/V'ci …… (47)

Las propiedades seudoreducidas se usan en la misma forma que las propiedades


reducidas de una sustancia pura:

PROBLEMA # 8
Una mezcla gaseosa tiene la siguiente composición: Eteno 57,00%, Argón
40,00%, Helio 3,00%. A 120 atm de presión y 25ºC el volumen determinado
experimentalmente es 0,144 L/molg. Comparar con el volumen molar calculado
por los siguientes métodos:
a) Gases ideales
b) Presiones parciales más el Z
c) Volúmenes parciales más el Z
d) Método de Kay
e) Valores medios de VDW
f) Calcular el % de error para cada método de (a) a (e).

SOLUCIÓN:
1,0 Base de cálculos: 1,000 molg

AMADO DE DIOS (amado_dedios@hotmail.com)[Escriba texto] Paá gina 69


Autor: Amado Crisógono Castro Chonta 2015-2
2,0 Por el método de gases ideales:
2.a) El volumen del gas será: V = RT/P

(0,082 057 L atm / mo lg k) (298,2º K )


V=

120 atm
V= 0,204 1/molg Rpta (a)

2.b) El % de Error será:


0,204  0,144
%E = 0,144
x 100 

%E = 41,7%

3,0 Por el método de Presiones parciales más el Z:

3.a) Construir la siguiente tabla para calcular Zm:


COMP Yj Pj Tj Pc Tc Prj Trj Zj Yj Zm
O- (atm) (ºK) (atm) (ºK) Zj
NENT
E
Eteno 0,5 68,40 298,2 50,50 283,1 1,35 1,053 0,38 0,218
7 0 0 4 3 3
Argón 0,4 48,00 298,2 47,94 150,6 1,00 1,979 0,98 0,329 0,640
0 0 5 1 0 0 5
Helio 0,0 3,60 298,2 2,26+ 5,26+ 0,35 22,49 1,00 0,030
3 0 8 8 1 0 5 2

3.b) Calcular el volumen molar (L/molg) usando la ecuación:


V = ZmRT/P  V = 0,640 5 (0,204 L/molg)
V = 0,131 L/molg Rpta (b)

3.c) El % de Error será:


 (0,144  0,131)
%E = x 100  %E = - 9,03% =  9,03%
0,144

4,0 Por el método de volúmenes parciales más el Z:


4.a) Construir la siguiente tabla para calcular Zm:

AMADO DE DIOS (amado_dedios@hotmail.com)[Escriba texto] Paá gina 70


Autor: Amado Crisógono Castro Chonta 2015-2
COMP Yj P T Pc Tc Prj Trj Zj Yj Zm
O- (atm) (ºK) (atm) (ºK) Zj
NENT
E
Eteno 0,5 120,0 298,2 50,50 283,1 2,38 1,053 0,38 0,218
7 0 0 0 0 3 3
Argón 0,4 120,0 298,2 47,94 150,6 150, 1,979 0,96 0,386 0,636
0 0 0 5 65 7 8 3
Helio 0,0 120,0 298,2 10,26 13,26 13,2 22,49 1,04 0,031
3 0 0 6 0 0 2

4.b) Calcular el volumen molar (L/molg) usando la ecuación:


V = ZmRT/P  V = 0,6363 (0,204 l/molg)
V = 0,130 L/molg Rpta (c)

4.c) El % de Error será:


 (0,144  0,130)
%E = x 100
0,144

%E = -9,72%=  9.72%

5,0 Por el método de Kay:


5.a) Construir la siguiente tabla para calcular Zm:
Yj Pc Tc Yj Pc Yj Tc p'r T´R
COMPONE (atm) (ºK) (atm) (K)
NTE
Eteno 0,57 50,50 283,10 28,79 161,40
Argón 0,40 47,94 150,65 19,18 60,30 2,49 1,34
Helio 0,03 10,26 13,26 0,31 0,40
P'c = 222,10 = T'c
48,27

P 120 atm
P´r =  P'r =
P' c 48,27 atm
P´r = 2,49
T 298,2 K
T´r =  T'r = 222,1K
T 'c
T´r = 1,34
Usar la gráfica #6 para estimar Zm = 0,70
V= 0,70(0,204 L/molg) entonces V= 0,143 L/molg Rpta (d)

AMADO DE DIOS (amado_dedios@hotmail.com)[Escriba texto] Paá gina 71


Autor: Amado Crisógono Castro Chonta 2015-2
 (0,144  0,131)
%E = x 100  %E = -0,70% =  0,70%
0,144

6,0 Por el método de los valores medios de VDW


6.a) Construir la siguiente tabla para calcular los valores medios de bm y am
Yj bj aj Yj bj (aj)1/2 Yj am
COMPONE (L/mol (l/molg)2a
NTE g) tm
Eteno 0,57 0,0572 4,48 0,032 60 1,206 46
Argón 0,40 0,0323 1,35 0,012 92 0,464 76 2,8115
Helio 0,03 0,0237 0,034 0,000 71 0,005 53
bm = 0,046 23 1,676 75 = am1/2

6.b) Calcular el V por el método de aproximaciones sucesivas. La ecuación de


estado de VDW para mezclas es:
am
(P + ) (V - bm) - RT = 0
V2
2,8115
(120 + ) (V - 0,04623) - 0,082057) (298,2) = 0
V2

Luego elaborar una tabla dando valores a V para calcular  , hasta que 
= 0  V es el valor buscado.

2,811 5
Vm (120 + ) (V – 0,046 23) - 24,470 3 
V2
0,20 190,288 0,153 77 24,470 3 4,79
0,15 244,95 0,103 77 24,470 3 0,95
0,14 263,44 0,093 77 24,470 3 0,23 Interpolando

V = 0,137 L/molg

6.c) El % de Error será:


(0,144  0,137)
%E = - x 100
0,144

%E = - 5,07% %E =  5,07

Algunas Ecuaciones Utilizadas


  mRT  RT
 Densidad molar:  m  . Masa molar  M    M  ….
M  VP  P

(21,4)

AMADO DE DIOS (amado_dedios@hotmail.com)[Escriba texto] Paá gina 72


Autor: Amado Crisógono Castro Chonta 2015-2
 Para calcular las propiedades del gas desalojado de un recipiente de volumen

RT
constante: V(P1 – P2) = (m1 – m2) …….(21,6)
M
a
 Van de Waals: ( P  ) (V - b) = RT
V2
RT a
 Clausius: P = -
V  b T (V  c )
2

RT a
 Lorentez: P = 2 (V +b) -
V V2

RT a
 Berthelot: P = -
V  b TV 2

RT A
 Keyes: P = - ; donde: S = e - /V
V S (V  1) 2

 Ecuación virial de estado.- Es una ecuación de la forma:


PV B (T ) C (T )
=1+ + + …….
RT V V2

AMADO DE DIOS (amado_dedios@hotmail.com)[Escriba texto] Paá gina 73


Autor: Amado Crisógono Castro Chonta 2015-2
 BIBLIOGRAFÍA

1. J. Smith y N. Van Ness. “Introduction to Chemical Engineering Theromodynacies”.


Editorial Mc Graw Hill. 7º Edición; 2003, México.
2. M. C. Reid y T. K. Sterwood. “The Proporties of Gases and Liquids. Editorial Mc
Graw Hill. 3º Edición; New York 1970.
3. D. Mimelblan. “Principios y Cálculos Básicos de la Ingeniería Química”. Editorial
CMCSA. 9º Edición; México 2003.
4. R. M. Perry y C. M. Chilton. “Chemical Engineer's Mandbook. Editorial Mc GRaw
Hill – USA. 5º Edición; USA 2001
5. L. C. Nelson y E. I. Obert. “Chemical Engeneering”. Vol 61; 7. Pag. 203 a 208
(1989)
6. G. J. Van Wylen y R. E. Sountag. “Fundamentos de Termodinámica”. Editorial
Limusa Wiley S.A. 7º Edición, México 2001.
7. Massermann y Otros. “propiedades termofísicas del Aire y de sus componentes”.
Editorial Nauta, 1966.
8. J. Aguilar Peris. “Termodinámica”. Editorial Alambra S.A. España - Madrid; 1981.

AMADO DE DIOS (amado_dedios@hotmail.com)[Escriba texto] Paá gina 74


Autor: Amado Crisógono Castro Chonta 2015-2

AMADO DE DIOS (amado_dedios@hotmail.com)[Escriba texto] Paá gina 75


Autor: Amado Crisógono Castro Chonta 2015-2

AMADO DE DIOS (amado_dedios@hotmail.com)[Escriba texto] Paá gina 76


Autor: Amado Crisógono Castro Chonta 2015-2

AMADO DE DIOS (amado_dedios@hotmail.com)[Escriba texto] Paá gina 77


Autor: Amado Crisógono Castro Chonta 2015-2

AMADO DE DIOS (amado_dedios@hotmail.com)[Escriba texto] Paá gina 78


Autor: Amado Crisógono Castro Chonta 2015-2
TABLA DE FACTORES DE CONVERSIÓN
MASA (M): unidad de medida en el SI (Kg)
1,0 UTM = 9,81 kg
1,0 slug = 32,2 lb
1,0 Kg = 1000 g = 103 g = 2,205 lb
1,0 lb = 16,0 oz = 453,6 g = 4,536 x 106 mg
1,0 ton = 2000 lb = 907,03 kg
1,0 TM = 1000 kg = 103 kg = 106 g = 109 mg
= 2205 lb = 68,5 slug = 101,95 UTM

LONGITUD (L): unidad de medida en el SI (m)


1,0 km = 1000 m = 103 m = 105 cm = 106 mm
1,0 m = 100 cm = 102 cm = 103 mm
= 1010 Aº = 106  = 109 m
= 39,37 plg = 3,2808 pie = 1,0936 yardas
1,0 cm = 108 Aº = 104  = 107 m
1,0 pie = 12 plg = 0,3048 m = 30,48 cm
= 304,8 mm
1,0 plg = 2,54 cm = 25,4 mm
1,0 yarda = 3,0 pie 91,44 cm
1,0 milla terrestre = 1609 m = 5 280 pie
1,0 milla marina = 1852 m = 6 076,3 pie
1,0 Aº = 10-8 cm = 10-10 m
1,0 u = 10-4 cm = 104 Aº

VOLUMEN (V): unidad de medida SI (m3)


1,0 m3 = 103 dm3 = 103 L = 1000 L
= 106 cm3 = 106 mL = 35,3147 pie3
= 264,172 gal us
1,0 pie3 = 0,028 317 m3 = 7,481 gal us
= 1 728 plg3 = 28,3168 L
1,0 bbl = 42 gal us = 5,615 pie3 = 159,0 L
1,0 gal us = 0,133 7 pie3 = 3,785 L

DENSIDAD (D) : unidad de medida en el SI (Kg/m3)


1.0 g/cm3 = 1000 kg/m3 = 103 kg/m3 = 103g/L
= 62,427 8 lb/pie3 = 3,612 7 x 10-2 lb/plg3
= 8,345 4 lb/gal

FUERZA (F) : unidad de medida en el SI (N)


1,0 kgf = 9,81 N = 9,81 Kg.m/s2
= 9,81 x 105 dyn = 103 gf
1,0 N = 105 dyn = 105 g.cm/s2
1,0 lbf = 4,448 2 N = 4,482 x 105 dyn = 32,174 lb.pie/s2

PRESIÓN (P) : unidad de medida en el SI (Pa)


1,0 atm = 33,91 pie H2O = 406,92 plg H2O
= 10,333 m H2O = 29,921 plg Hg

AMADO DE DIOS (amado_dedios@hotmail.com)[Escriba texto] Paá gina 79


Autor: Amado Crisógono Castro Chonta 2015-2
= 76,0 cm Hg = 760 mm Hg
= 1,033 3 kgf/cm2 = 103 33 Kgf /m2
2
= 14.696 Lbf/plg abs = 14.696 psia
= 1,013 25 bar = 101 325 N/m2
= 101 325 Pa = 1 013 250 dyn/cm2
1,0 bar = 105 N/m =10 Pa = 106 dym/cm2
2 5

= 750,061 mmHg = 14,503 8 psia


1,0 kgf/cm2 = 735 mmHg = 9,81 x 104 Pa = 10 m H2O
1,0 mm H2O 9,81 Pa =
1,0 mmHg = 133,3 Pa =
1,0 lbf/pie2 = 47,88 Pa = 47,88 x 10-5 bar
1,0 lbf/plg2 = 6 895 Pa = 0,068 95 bar

ENERGÍA: unidad de medida en el SI (J)


1,0 J = 1,0 mN = 1,0 W.s = 1,0 kg.m2/s2
= 107 ergio = 107 dyn. cm
3
= 10 cm bar = 102 L.bar = 10-5m3 bar
3
= 9,869 23 cm .atm = 9,869 23 x 10-5 m3 bar
-3 3
= 5,121 97 x 10 psia.pie = 0,239 006 cal
= 0,737562 pie.lbf = 9,478 31 x 10-4 Btu
1,0 cal = 4,184 J = 4,184 x 107 erg
= 41.84 cm3 bar = 0.041 84 L.bar
= 41,293 cm3.atm = 0,041 293 L.atm
= 0,021 43 psia.pie3 = 3,086 1 pie.lbf
1,0 kcal = 103 cal = 4 184 J
1,0 Btu = 1055 J = 778,16 pie.lbf = 252,15 cal
= 10,412 L.atm = 107,587 m.Kgf
1,0 L.atm = 10,333 m kgf = 24,214 cal
1,0 pie.lbf = 12,0 Plg.lbf = 1,355 82 J = 0,324 05 cal
1,0 m. kgf = 9,806 J
1,0 ev = 1,602 x 10-12 erg = 10-6 Mev

POTENCIA: unidad de medida en el SI (W)


1,0 kw = 103W = 103 J/s = 103 VA = 103 kg.m/s3
= 3,6 x 106 J/h = 239,006 cal/s
= 737,562 pie.lbf.s-1 = 860,4 kcal/s
= 56,869 9 Btu,min-1 = 1,341 02 HP
= 3412,2 Btu.hr-1
1,0 HP = 2544,5 Btu.hr-1 = 745,7 W = 745,7 J/s
= 550 pie.lbf/s = 33 000 pie.lbf/min
= 760,0 m.kgf/s
1,0 CV = 735,7 w = 735,7 J/s = 633 kcal/h
1.0 Btu/s = 1,055 kw

CONSTANTE UNIVERSAL DE LOS GASES IDEALES (R)


R = 0,082057 l.atm/molg K = 82,057 cm3.atm/molg K
= 8,317 J/molg ºK = 8,317 x 107 erg/molg K = 8,314 4 m3.Pa/molg K
= 1,987 cal/molg ºK = 1,987 Btu/mol lb R
= 21,9 (plg Hg) pie3/mol lb R = 0,083 144 L.bar/molg K
= 62,361 L.mmHg/molg K = 0,730 2 pie3.psia/mol lb R
= 10,731 pie3.psia/mol lb R = 0,0848 (l) (kgf cm-2)/molg K
= 998,9 pie3.mmHg/mol lb K = 1,314 pie3.atm/mol lb K

AMADO DE DIOS (amado_dedios@hotmail.com)[Escriba texto] Paá gina 80


Autor: Amado Crisógono Castro Chonta 2015-2
= 555 pie3.atm/mol lb R = 1 545 pie.lbf/mol lb R
= 7,805 x 10-4 hp.hr/mol lb R = 5,819 x 10-4 kw.hr/mol lb ºR
= 1,851 x 104 plg lbf/mol lb ºR=
= 44,568 (pie H2O) (pie3)/mol lb ºK

CONSTANTES:
Gravedad (g) y gc
9,8066 m/s2 98 066 kg.m/kgf.s2
980,66 cm/s2 32,174 lb.pie/lbf.s2
32,174 pie/s2 1,0 kg.m/Ns2
1,0 g.cm/dym.s2
CAPACIDAD CALORÍFICA : unidades en el SI (J/mol.K)
Energía especifica: unidades en el SI (J/Kg)
1,0 cal/g K = 1.0 Btu/lb ºF = 4,184 J/g ºC
Capacidad calorífica molar
1,0 cal/molg K = 1,0 Btu/mol lb ºR = 4,184 J/molg ºK
Calor y entalpia especifica:
1,0 Btu/lb = 0,5559 cal/g = 2 325,89 J/kg
1,0 cal/g = 4,184 J/g = 4 184 J/kg = 1,8 Btu/lb
Viscosidad absoluta: unidades en el SI (Pa.s)
1,0 Pa.s = 1,0 N.s/m2 = 10 dym.s/cm2
= 10 p = 0,672 1 lb/pie.s = 0,102 kgf.s/m2
1.0 p = 1,0 g/cm.s = 0,1 kg/m.s
1,0 cp = 0,01 p = 2.4195 lb/pie.h
VISCOSIDAD CINEMÁTICA: unidades en el SI (m2/s)
1,0 st = 1,0 cm2/s = 10-4 m2/s
1,0 m2/s = 10,763 9 pie2/s
CONDUCTIVIDAD TÉRMICA: unidades SI (w/ms)
1,0 Btu/pie.h.ºF = 1,729 7 W/mk
= 1,488 3 kcal/m.h. ºC
1,0 kcal/m.h. ºC = 1,162 2 W/mk

AMADO DE DIOS (amado_dedios@hotmail.com)[Escriba texto] Paá gina 81


Autor: Amado Crisógono Castro Chonta 2015-2
COEFICIENTE DE PELICULA O COEFICIENTE DE TRANSFERENCIA DE CALOR
w/m2k
1,0 kcal/m2 hº C = 1,162 2 W/m2k
1,0 Btu/pie2 hº F = 5,675 W/m2k = 4.8830 kcal/m2 h ºC

UTM  unidad técnica de masa en el sistema Inglés


Slug  unidad técnica de masa en el sistema británico
Ton  tonelada corta. TM  tonelada métrica. plg  pulgada
bbl  barril. gal us  galón americano. lb  libra masa
lbf  libra peso. dym  DINA. Pa  pasacal.
J  youl. W  watt. U  micro
HP  caballo de fuerza. CV  caballo de vapor

AMADO DE DIOS (amado_dedios@hotmail.com)[Escriba texto] Paá gina 82


Autor: Amado Crisógono Castro Chonta 2015-2
PROBLEMAS

1. Calcular las presiones dadas por las ecuaciones de un gas ideal y de Van Der Waals
para 10 mol de CO2 que ocupa un volumen de 3,81 x 10 -3 m3 a 40ºC. Si el vapor
experimental es de 50 atm; calcular el % de error para cada caso?
2. Calcular los volúmenes por las ecuaciones de estado de los gases ideales, Van Der
Waals y del factor de compresibilidad para 5 mol de N2 a 400 atm y 0ºC. Comparar
y calcular los % de error, si el valor experimental es 3,515 x 10-4 m3.
3. La ecuación de Berthelot es (P + a/Tv2) (v-b) = RT. Demostrar:
i) a = (16/3) PcTcV2c ; ii) b = (1/4) Vc
9 PTc
iii) Pv = RT (1 + (1-6T2c/T2))
128 Pc T

4. Demostrar que a presiones moderadas y bajas la ecuación de VDW se puede escribir


en la forma Pv = RT (1-BP); donde B = (1/RT) (a/RT-b). sugerencias: despreciar el
término ab/v2 que es pequeño si la presión no es grande y sustituir v = RT/P en a/v.
5. Un cilindro de 100,0 x 10-3 m3 de capacidad contiene gas metano a 200,0 atm y 25ºC
Determinar el peso de gas gastado cuando la presión en el cilindro ha caído a 50,0
atm, por tres métodos: a) gas ideal, b) VDW, y c) factor Z.
6. Usando los datos de temperatura y presión crítica del etano calcular las constantes
de VDW y de Berthelot. Cuál sería el volumen molar crítico y comparar con el
vapor experimental?
7. Calcular los volúmenes por: i) gas ideal; ii) VDW y iii) factor de compresibilidad
para 5.5 mol de H2 a 500 atm y 250 K.
8. Una mezcla formada por 46,0 g de O 2 y 154,0 g de N2 ocupan 310 x 10-3 m3 a
273.15 ºK. Calcular la presión por los métodos de los: i) gases ideales, ii) VDW +
Dalton; iii) VDW + Amaget; iiii) VDW y las constantes medias; iiiii) Factor Z +
Amagat; 6i) Factor Z + Dalton y 7i) Kay.
9. Hallar la forma reducida de la ecuación de estado de Berthelot: (P+a/Tv2) (v-b) ) RT.
10. Determinar las constantes críticas de un gas que obedece la siguiente ecuación de
estado: P = RT/(v-b)-a/T(v-c)2.
11. Un gas real posee las siguientes densidades a 300,0 K;
Presión P(atm) 0,40 0,80 1,00
Densidad (g/l) 1,512 3,088 3,900
Calcular el peso molecular del gas?

AMADO DE DIOS (amado_dedios@hotmail.com)[Escriba texto] Paá gina 83


Autor: Amado Crisógono Castro Chonta 2015-2
12. Usando los parámetros críticos Pc y Tc determinar los valores de las constantes a y b
de las ecuaciones de estado de berthelot y VDW de los siguientes gases: a) agua; b)
SO2; c) etanol d) amoniaco; e) CS2; f) CCl4 y g) C6H6.
13. Hallar los valores de las constantes a , b y c de la ecuación empírica de Wohl;
usando los parámetros críticos Pc y Tc de los gases propuestos en el problema 12.
14. Determinar los valores de las constantes a y b de la ecuación de estado de Dieterici y
representar ésta en forma reducida.
15. Calcular la densidad molar de la siguiente mezca gaseosa a 600,3 psig y 300ºF:
metano 100 lb; etano 240 lb; propano 150 lb; y N2 50 lb; mediante los métodos
propuestos en el problema 8.
16. El análisis de un gas indica 50,5% de metano y el resto etileno a 1 atm. Se desea
almacenar 25 kg de esta mezcla gaseosa en un cilindro cuya capacidad es de 3,26
pie3 a una temperatura máxima de 320 K. Calcular la presión dentro del cilindro por
los siguientes métodos: a) gas ideal; b) VDW y las constantes medias; c) factor Z +
Amagat; d) Kay. Cuántas libras de esta mezcla pueden almacenarse en el cilindro a
320 K si la máxima presión permisible es de 1350 psig?
17. Se va llevar gas natural por una tubería de 30,54cm de diámetro interno. El metano
puro entra al gaseoducto a razón de 70 lb/s a una presión de 3,000 lb f/plg2 y a una
temperatura de 70ºF. Calcular la densidad incivil en el Si suponiendo que el metano
obedece a la ecuación de estado de: a) gas ideal; b) VDW; y c) factor Z.

AMADO DE DIOS (amado_dedios@hotmail.com)[Escriba texto] Paá gina 84


Autor: Amado Crisógono Castro Chonta 2015-2

AMADO DE DIOS (amado_dedios@hotmail.com)[Escriba texto] Paá gina 85

S-ar putea să vă placă și